Sei sulla pagina 1di 170

MBChB Year 4 & 5 Past Papers + Answers

Courtesy of PALI, Tina Bylinski, Sophie Coyle & Hannah Gower

Question 1: Respiratory
A 42 year old man is admitted to hospital with left sided pleuritic chest pain with
haemoptysis and crepitations in the left base. You suspect he has a diagnosis of
pneumonia. He tells you that 3 months beforehand he was diagnosed as having a
carcinoma of the lung and he has been receiving chemotherapy.

There are 4 different varieties of malignant lung tumors. Please list 2.

List 4 common presenting symptoms of lung carcinoma.

What would your first line treatment for the community acquired pneumonia?

Name 2 measures that can used to increase sputum production.

As part of your investigation you discover that his platelet count in 75. List 3 possible
causes.

Question 2: Gastrointestinal/ Hepatic


An 83 year old woman presents with a 2 week history of obstructive jaundice. She has
been diabetic for 2 months and has lost 7Kg. She has pale stools and dark urine.

What 2 initial investigations would your organise?

What are the 2 main diagnoses consider?

Investigations revealed a dilated biliary tree and ERCP is planned. Name 3 potential
complications of this procedure.

Cytology reveals malignant cells; what 3 management options would you like to
discuss with the patient?

Question 3: Respiratory
A 23 year old asthmatic is brought into the A&E Department with an acute
exacerbation. He has become increasingly short of breath over the last three weeks.
You make a diagnosis of acute asthma attack.

Give 2 clinical factors you would wish to establish in the history from this patient in
order to assess the severity of her attack.

Give 4 clinical factors you should establish in the examination of the patient in order
to assess the severity of her attack.

What immediate investigations might usually be performed in A&E and what


abnormality in each would cause you concern. List three.

What 3 categories of treatment would consider for this patient?

Question 4: Infection
An 18 year old university student is seen by his GP with a 24 hour history of flu like
illness, fever headache and neck stiffness. He is noted to have a progressively
purpuric rash. There are no known drug allergies.

What is the most likely diagnosis?

Give 2 examples of appropriate antibiotics which should be administered immediately


by the GP and which route?

How might a positive microbiological diagnosis be made? Suggest 4 tests,

Name 2 public health implications of the suspected diagnosis?

What is the main limitation of the currently available vaccine for this condition?

Question 5: Orthopaedics/ MSK


A 68 year old lady underwent total hip replacement for osteoarthritis of the left hp. The
operative procedure was uneventful. She has increasing pain and swelling in her calf 3
days following surgery. Clinical examination revealed swelling of the left leg and foot
but there is no colour change.

What is the diagnosis?

Name any important causes in the ladys case.

What are the 3 factors which influence venous thrombosis, known as Virchows triad?

Name any 2 prophylactic measures that are recommended to prevent this problem
after total hip replacement.

What is the worst complication that can result from this problem?

Question 6: Breast
A 57 year old teacher is seen in the breast clinic. She has been aware of a mass in her
left breast for 2 weeks. She is worried about cancer.

What clinical features may suggest that the lesion may be malignant? List 3.

Name 2 investigations which will help establish the diagnosis of breast carcinoma.

Name 2 main surgical approaches to treating breast carcinoma.

List 3 pieces of code histological information required from the pathologist that will be
needed by the oncology team to decide further treatment.

Question 7: Neurology
While working as a FYI on a medical ward you are asked to asses Mr FK,
a 75 year old woman who was admitted to hospital one week previously
with a sudden onset of weakness in the right arm and leg. On
examination you confirm the weakness and also find that the muscle
tone in the right arm and leg is increased. Sensation is on the right
side Although She can talk, she sometimes has difficulty finding the
words she wants.

What changes do you expect in the tendon reflexes on the right leg?

What is the mechanism of this alteration to the reflexes?

What do you expect the right plantar reflex to be?

Which cranial n. is the most likely one to be affected?

What visual field abnormality might you expect to find on examination?

Over the next 24 hours the patient's condition deteriorates. A CT scan confirms an
infarct in the left middle cerebral a. territory, Her husband calls you aside and ask you
to write in her notes that she should not be resuscitated if she stops breathing.

Which two articles of the Human Rights Act are most applicable when considering
these issues?

Having established the diagnosis, what three issues do you need to take into
consideration before writing a Do Not Resuscitate order?
8

Question 8: Neurology
A 68 year old man presents to the medical clinic. He describes a several year history
of gradually reducing mobility and failure to cope at home. You examine the patient,
and diagnose Parkinsonism.

What are the cardinal features of Parkinsonism on examination?

Name 3 possible causes of Parkinsonism.

You decide to treat the Parkinsonism with Dopamine agonists. Apart from nausea and
GI upsets, list 2 common side effects of treatment using L-DOPA.

How can these side effects be minimised?

Name 2 other drugs used to treat Parkinsonism.

10

Question 9: Gastrointestinal/ Hepatic


Mrs R.S. is a 75 year old, previously well, woman who has been admitted to hospital
under your care as an emergency with a history of two hours of severe upper
abdominal pain and vomiting. On examination she is obviously distressed with a
tachycardia but is otherwise hemodynamically stable. The abdomen is tender with
guarding in the upper part. Bowel sounds are diminished. There are no other relevant
findings.

List 3 important likely diagnoses.

What key early investigations may help you resolve the differential diagnosis? List 3.

After the patients initial assessment, but before definitive treatment, what 4 urgent
measures would you institute?

11

Question 10: Gastrointestinal/ Hepatic


A 39 year old woman is admitted to the medical ward from the GPs surgery with a 2
hour history of vomiting fresh blood. She is pale with cold peripheries. Pulse 120/min,
BP 80/46 mmHg. Her breathing is satisfactory.

List 3 common causes of severe upper GI blood loss.

As the FY1 you take 20 mls of blood and request a cross-match, blood count,
electrolytes and clotting studies. You also take a brief history and perform a clinical
examination.

Apart from the above, suggest 4 steps you would take in your initial management of
this patient in the first 15 minutes after arrival on the ward.

Once the patient is stable, list 3 monitoring instructions that you would ask the nurses
to carry out on the patients behalf.

12

Question 11: Gastrointestinal/ Hepatic


A 70 year old man presents with a recent history of rectal bleeding and a change in
bowel habits so that he is now more constipated than usual. He has no past medical
or surgical history and rectal examination reveals a hard mass 6 cm from the anal
verge.

What is the most likely diagnosis?

Give 4 other possible causes of rectal bleeding in a 70 year old man.

What urgent investigation is required to confirm your likely diagnosis?

If the likely diagnosis is confirmed, suggest 4 additional investigations which you


would like to undertake and give a short reason for each.

13

Question 12: Neurology


An 81 year old woman is found collapsed on her bedroom floor by the sheltered
housing warden the day after a trip to her bingo. A CT scan of the brain reveals an
area of ischaemia in the left parietal cortex, consistent with a recent cerebral infarct.
She is badly bruised and has an obvious weakness on the right side of her body. She
is confused and her speech sounds slurred.

What is the definition of a stroke?


A rapidly developing focal neurological deficit of vascular origin lasting over 24 hours or
resulting in death.

Suggest 4 risk factors for stroke.


age
hypertension
atherosclerosis
aneurysms
AF
DM
smoking
Previous TIA/stroke
OCP use
coagulopathy
sedentary lifestyle
hypercholesterolaemia
Ht
cocaine use

asian descent

A CT scan of the brain reveals an area of ischaemia. Explain the pathogenesis of this
cause of stroke.
Narrowing: Narrowing of the supplying blood vessels (thrombus or embolus) causes reduced
blood flow (and thus oxygen and glucose) to an area of the brain
Penumbra: There is a central area of necrosis surrounded by a penumbra that may be
salvageable if blood supply is reestablished
Ischemic Cascade: The ischaemic cascade is initiated causing inflammation and oedema
that results in tissue damage
Destructive Enzymes:This leads to glutamate toxicity and cell membrane permeability
changes thus activating destructive enzymes

Given this ladys symptoms, which is the most likely artery to have been affected by
this stroke?
left middle cerebral a.

List 4 significant non-neurological complications of stroke.


14

Aspiration pneumonia
DVT/PE due to immobility
Communication difficulties due to dysphasia and dysarthria
Depression
Bed sores due to immobility
This patient shows minimal improvement over the next three months.

Outline 2 management options that the OT would be able to help with in cases like
this.
Home assessment and adaptations where appropriate
Physical and cognitive deficit screen and provision of aids where needed

15

Question 13: Orthopaedics/ MSK


A 35 year old manual labourer was lifting a heavy box two weeks ago when he noticed
sudden sharp low back pain. The pain increases on coughing or sneezing and radiates
to his left leg and down to his toes. He has numbness of the dorsum of the left foot
and cannot extend his toes. Forward flexion of lumbar spine is markedly restricted.
You suspect a prolapsed intervertebral disc.

From the history and signs, which disc would be involved and which nerve root is
being compressed?
L5

What abnormality would you expect to see when examining this patients knee and
ankle reflexes?
Both would be present
The ankle reflex is controlled by S1 so would only be lost by lesion there
The knee jerk is mainly controlled by L4 so would only be lost by a lesion there

What 2 sign and symptom combinations might suggest a central disc prolapse?
Bilateral Leg Pain / Weakness
Urinary Retention / Incontinence
Perianal / Perineal Sensory Loss
Reduced Anal Tone

With the patient in a supine position, what test would you perform to help establish
the diagnosis and what would this show?
Test Straight leg raise
Result Limitation of straight leg raising with sciatica pain radiating down the
buttock and lower limb

Name 3 drugs used for initial treatment of this patient.


NSAIDs - topical + oral eg ibuprofen
co-codamol if escalation of analgesia
paracetamol
gabapentin if n. pain

Which investigation might you consider to confirm the diagnosis?


MRI Lumbosacral spine

What can paramedical staff offer to support treatment in this condition?


Behavioural therapy
Occupational T can get back schools in occupational setting
Physiotherapy staying active recommended

What surgical treatment could be considered?


discectomy

In <50 words define what surgical treatment is appropriate and explain what
proportion of patients are likely to require surgery.
16

Discectomy is the surgical removal of herniated disc material that presses on a nerve root.
Microdiscectomy is a minimally invasive procedure in which a portion of a herniated nucleus
pulposus is removed by laser while using a microscope.
90% resolve at 8 weeks with analgesia. 1 year outcomes are the same in those who are
managed conservatively and those who get surgery. Therefore only 10% of patients receive
surgery. Also surgery + intensive exercise programme leads to significantly improved
functional status and faster return to normal.

Name 2 local complications of surgical treatment.


Relieved sciatica but continuing back pain
Nerve damage

17

Question 14: Endocrinology


A 47 year old medical secretary is seen in clinic complaining of weight loss, sweating
and palpitations. She has no previous relevant history. On examination you detect a
symmetrically enlarged thyroid gland.
Investigations reveal a T4 of 75 nmol L-1 (reference range 10-25 nmol L-1) and TSH <
0.01 molL-1 (reference range 0.2-5 molL-1).

What is the most likely pathological mechanism causing thyrotoxicosis in this


instance?
Graves Disease Autoimmune condition resulting in production of IgG TSH
Receptor
Autoantibodies overstimulation of thyroid hormone

Name 8 other signs you might detect on examination.


Fine tremor
Palmar erythema/warm, sweaty hands
Tachycardia (may be irregular if AF)
Lid retraction
Lid lag
Exophthalmos
Tachycardia

Apart from beta-blockers, list 2 drugs that are commonly used for medical
management of thyrotoxicosis.
Carbimazole
Polythiouracil

What is the most serious side-effect that a patient may develop on anti thyroid drugs?
Bone Marrow Suppression, which may lead to pancytopenia and agranulocytosis

What common clinical condition would you tell the patient to be aware of that may
alert one to the development of this condition?
Tell the patient to seek medical advice immediately if they develop bruising, mouth ulcers,
sore throat, fever, malaise, or non-specific illness

What options are available for more definitive management of thyrotoxicosis? Name
two
radioiodine therapy
thyroidectomy

18

Question 15: Infection


An 18 year old Sociology student presents to Accident and Emergency with witnessed
episodes of arm and leg jerking. Although he is maintaining his airway without
assistance, he is drowsy but responds to commands and you suspect he is post ictal.
His girlfriend states that he had a nonspecific viral illness for two days prior to
presentation but he is not on medication, and has no past medical history.

Apart from infection, suggest 6 possible causes for his seizures.


epilepsy
alcohol
substance abuse
tumors (SoL)
Trauma (HI)
Hypoglycemia
Hyponatremia
Stroke
Hypoxia
Syncope
He remains drowsy and appears to have increased tone in his left arm with brisk
reflexes but assessment of airway breathing and circulation shows no abnormality. He
doesnt let you switch on the light, and his neck is very stiff. Nurse mentions his
temperature is 38.7oC. You suspect he has developed bacterial meningitis.

What action should be taken immediately?


IV antibiotics (in adults Ceftriaxone 2g)

Aside from routine blood tests (FBC, U&E, LFTs), list 3 other parts of your
management plan (including treatment and investigations).
CT Brain - No LP due to possible raised ICP indicated by seizure
Blood cultures and PCR for Neisseria
Coagulation Screen
IV fluids
Antipyrexia
Septic Screen
Contact HDU or ITU

Name 4 common complications of this illness.


Septic shock
Hearing loss
Seizures
Intellectual impairment

Bearing in mind his occupation, which other agency should be informed and why?
Public Health
Bacterial Meningitis is a notifiable disease so his contacts will need to be traced and treated

19

Question 16: Gastrointestinal/ Hepatic


A 25 year old accountant is admitted as an emergency under your care with a one
week history diarrhoea with blood through the stool. He has no previous relevant
history. Abdominal examination is unremarkable.

Name 3 common causes of these symptoms.


Crohns Disease
Ulcerative Colitis
Infectious Colitis

What investigations could help clarify the diagnosis and its underlying cause? Name
3.
AXR - rule out any masses, faecal loading
Colonoscopy - investigation lower GI tract
Biopsy + histology - determine presence of colitis

Biopsies suggest mucosal inflammation with crypt abscesses. What is the most likely
diagnosis?
Crohns disease

Which classes of drug might be used in treating this condition? Name two.
5-ASA
oral steroid therapy 40mg OD - then taper down to a balance between symptoms & lowest
dose before relapse

If the patient deteriorates despite drug treatment, what urgent operation might be
appropriate?
colectomy

20

Question 17: Gastrointestinal/ Hepatic


A 28 year old man presents with a two month history of increasingly frequent bowel
actions. He has been gradually losing weight. There is no relevant past medical
history. Examination reveals a 55kg man of normal height who looks pale and anxious.
There are no abnormal abdominal findings.

What additional history is it important to obtain from the patient at this stage? List 4
items.
presence of PR blood?
any skin changes (pyoderma gangrenosum, erythema nodosum)?
frequency of stool motions?
any nausea/ vomiting +/- haematemesis?
any uveitis, iritis, conjunctivitis?
any PMHx spondyloarthropathies?
any systemic features: SoB, c, night sweats, Kg , loss of appetite, thirst?
any FHx: gluten enteropathy/ coeliac disease, IBD (Crohns, Ulcerative Colitis), CRC, FAP,
HNPCC?
any recent travel/ any unwell contacts?
any HIV risk factors?

As the patients General Practitioner you would like to perform some simple
investigations (before considering referral for a specialist opinion). List 2.
Digital Rectal Exam
Blood tests -> FBC, ESR, CRP
Coeliac screen
Faecal calprotectin
Stool Culture/ Microscopy & C.Difficile test
Ova & Parasite Exam
Thyroid Function Tests
U&Es - degree of dehydration
HIV test
CEA level (unlikely in younger patients but if FHx)

The patient asks why he might be losing weight and you wonder about malabsorption.
Give two tests that could help you identify whether the patient is suffering from
malabsorption.
Vitamin B12 serum level
Folate & Ferritin levels
INR

You decide to request a Specialist opinion. Your local Gastroenterologist arranges a


gastroscopy and a duodenal biopsy reveals flattening and irregularity of the villous
architecture, crypt hyperplasia and raised numbers of intraepithelial lymphocytes.
What is the most likely diagnosis in this case?
Celiac Disease

What blood tests can be used to test for this condition?


serum IgA level, as about 2-3% of coeliac disease patients are IgA-deficient
IgG anti-TTG and/or IgG EMA if IgA deficiency is confirmed

21

Question 18: Cardiovascular


A 32 year old woman, who is a known alcoholic and abuser of intravenous drugs,
presents to Accident & Emergency complaining of gradual onset of malaise, fever,
weight loss and night sweats. She is pyrexial (38.5 oC). She has a pansystolic murmur
which is thought to be a new finding and you suspect she has a diagnosis of infective
endocarditis.

Name 4 additional clinical signs that may be found on examination in this patient.
Murmur
Anaemia
Abscess
Clubbing
Roth Spots
Oslers Nodes
Splinter Haemorrhages
Splenomegaly
Janeways Lesions
Haematuria
Petechiae

Name the 2 most likely organisms to be implicated in infective endocarditis.


streptococcus viridans
staphylococcus aureus

Your FY2 asks you to test the urine. What would you expect to find and what is the
pathology behind this abnormality?
haematuria - glomerulonephritis or renal infarct
On further examination you can also hear the pansystolic murmur. This is loudest at
the left sternal edge and you demonstrate her JVP is elevated with giant v waves. In
addition she also has tender pulsatile hepatomegaly.

What is the most likely cardiac lesion to be responsible for this, given the above
history and examination?
tricuspid regurgitation

Name 2 investigations that are mandatory to confirm the clinical diagnosis of infective
endocarditis.
blood cultures - 3 sets at different times from different places
TTE Echo - demonstrating vegetation on affected valves

Other than intravenous drug abuse, name 4 other risk factors for infective
endocarditis.
22

Dental Surgery
Prosthetic Heart Valve
Thoracotomy
Pre-existing Valvular Disease i.e. Rheumatic, Congenital, Acquired
Catheterisation
Peripheral/Central Lines
Immunosuppression

23

Question 19: Cardiovascular


A 72-year old lady has been treated for mild heart failure for a number of years. She is
admitted to hospital as an emergency one night with a 48 hour history of worsening
shortness of breath. On examination you find her to be severely unwell, coughing pink
frothy sputum, with a marked tachycardia and profuse fine crackles at both lung
bases. No murmurs are audible. You make a rapid initial diagnosis of left ventricular
failure.

What 2 immediate interventions would you make?


sit her up
give 100% O2 through facemask

Name 2 drugs which may be helpful.


thiazide diuretic/Furosemide - to increase fluid excretion
Glyceryl Trinitrate

You arrange a chest X-ray. What 3 abnormalities would support your diagnosis?
Alveolar edema
Kerley B lines/ interstitial odema
Cardiomegaly
Dilated Prominent Upper Lobe vessels
pleural effusion

She improves and you are now able to hear a pansystolic murmur at the apex. What
cardiac lesion is likely to be responsible for this?
mitral regurgitation

List 3 possible causes for her clinical deterioration.


MI
PE
Cardiac tamponade
rupture of IV septum causing a VSD
AF

24

Question 20: Neurology


Mrs RS is an active 70 year old woman with atrial fibrillation. As the FY1 you see her
when she attends the Emergency Department with a complaint of loss of vision in the
left eye, unaccompanied by pain. She thinks she may have had previous episodes that
recovered and that the symptoms came on over a period of less than 30 minutes. She
has not experienced associated headaches.

What is the most likely cause of these symptoms?


retinal a occlusion

Name 2 other causes of sudden loss of vision in one eye


Retinal Artery Occlusion (Sudden)
Retinal Vein Occlusion (Sudden)
Retinal Detachment (Sudden)
Optic Neuropathy (esp. non-arteritic ischaemic ON which causes sudden painless loss of
VA)
Diabetic Retinopathy (vitreous haemorrhage would cause sudden loss, but most retinopathy
causes gradual loss)

What 2 points from the history, as given above, help you to distinguish between the
possible causes of vision loss in this patient?
absence of headache - typical presentation of GCA typically involves headache
lasting under 30 minutes - narrows differential, vascular cause more likely
AF -> risk of retinal a. occlusion due to embolus

What features of the ophthalmic examination would be important for you to note in
this patient? List 4 points
threadlike arterioles
prominent fovea (cherry red spot at macka)
pallor fundus
visual acuity (<6/60)
afferent pupil defect
cattle trucking in retinal arterioles (segmentation of blood column in the arterioles)

What investigation would you perform with regard to the carotid artery?
CT angiography - atherosclerostic plaques, stenosis

25

Question 21: GU
A 69 year old man presents to Emergency Department with severe lumbar back pain,
which has been increasing for some months. There is no history of injury. He has not
seen a doctor for many years. There are no neurological symptoms. A spinal X-ray
reveals multiple sclerotic lesions in the lumbar spine suggestive of metastatic
prostatic cancer.

List 2 investigations you would wish to carry out to investigate the prostate
enlargement.
PSA
Transrectal USS-guided Biopsy

List 2 investigations you would wish to carry out to investigate the degree of
metastasis.
Staging CT Chest/Abdo/Pelvis
Bone scan look specifically for other bony metastases
Prostascint scan to look for soft-tissue metastases is NOT a valid answer as it is very rarely
done

What is the most common type of malignant tumour occurring in the prostate gland?
adenocarcinoma

The diagnosis is confirmed and the patient is deemed unsuitable for lumbar spine
surgery. What other treatment options should be considered for this patient? List
three.
Androgen Ablation Hormonal (GnRH Receptor Antagonists or Surgical e.g.
orchidectomy)
Palliative Chemotherapy
Palliative Radiotherapy to bony metastases (external beam)

Explain the difference between stage and grade in the pathological/clinical


assessment of malignant tumours.
grade reflects the degree of mitotic abnormalities detected within the cancerous cells
stage is a reflection of a number of prognostic risk factors that reflect patient outcomes in
terms of mortality and morbidity eg spread (TNM)

26

Question 22: Gastrointestinal/ Hepatic


A 75 year old man is admitted as an emergency under your care with a one-day history
of severe generalised abdominal pain. He has no previous relevant history. On
examination he is shocked and distressed. His abdomen is rigid, diffusely tender and
silent and a chest X-ray suggests free intraperitoneal air.

What is the most likely diagnosis?


sepsis 2* to bowel perforation

Name 3 potential causes of this condition.


constipation
diverticulitis
mural malignancy

What initial therapeutic measures should be instituted? (Name 3)


sepsis 6 - depending on whether observations indicate likelihood
NGT if obstruction suspected
ABCDE approach
analgesia - 1mg Paracetamol O PRN (<4mg)
surgical review - discuss diagnostic laparotomy

What investigation might you request to support your clinical diagnosis?


CT scan abdomen - detect perforation of bowel

The patient underwent a laparotomy. What aims would surgery attempt to achieve?
(Name 2)
close perforation, bowel washout and restore bowel functionability
assess the presence of any contributing factors eg malignancy

27

Question 23: Orthopaedics/ MSK


A 38-year old man presents to the Orthopaedic Outpatient Clinic with a six month
history of pain in the right groin. Examination reveals mild restriction of right hip
movements in all directions. You suspect avascular necrosis of the femoral head.

Name 2 common causes of avascular necrosis of femoral head.


Trauma
idiopathic especially middle aged men affecting hips & knees

Name any 3 investigations in the diagnosis and evaluation of avascular necrosis of the
femoral head.
XR Both hips - assess severity of damage, any other pathology, whether the left joint is also
affected

Name 2 other areas of the skeleton that may be affected by a traumatic avascular
necrosis.
proximal pole of scaphoid bone
body of talus

What is an early radiological feature of hip avascular necrosis?


none
trabeculae with sclerosis
osteolysis areas

What is a late radiological manifestation of hip avascular necrosis?


crescent sign
osteochondral fracture
flattening of femoral head

What surgical procedure is recommended in the late stage of the disease?


arthroplasty

28

Question 24: Respiratory


A 62 year old housewife, who has never worked outside the home, has a history of
four months weight loss and more recently breathlessness. You are working as a FY1
on the admissions unit. On examination of the chest she has an area of right-sided
dullness to percussion.

You consider that this is a pleural effusion. What would you expect to see on a plain
chest X-ray? (Give 2 features)
blunting of costophrenic angles
opaque consolidation in right hemithorax with concave meniscus sign

You consider performing a diagnostic aspiration. What additional imaging


investigation would be of use prior to the aspiration?
Ultrasound (pleural)

You proceed with the pleural aspiration. What position should the patient ideally adopt
in order to perform the aspiration?
on the bed, slightly rotated with arm on the affected side elevated in order to expose the
axillary area

What 3 main diagnoses would you consider?


pleural effusion
haemothorax
empyema

When you withdraw the fluid with a diagnostic tap it is clear and has a yellow
colouration. What 3 important investigations would you request and why?
LDH and Protein in Pleural Fluid/Serum determine whether effusion is exudate
or transudate
Effusion Cytology to investigate possible concerns of malignancy
Glucose or pH of Effusion investigate possible malignancy (both should be low)
Her husband comes into the ward to see you.

He tells you that he worked in the shipyards for 25 years before he retired. Why would
this be relevant?
His wife may have been exposed to asbestos through him increased risk of
pleural mesothelioma

29

Question 25: Haematology


You are on cover for a surgical ward when you are called to Mrs McCulloch, who has
just commenced a blood transfusion for a low postoperative haemoglobin. Three
minutes after the start of the transfusion the patient develops lumbar pains, rigors,
dyspnoea and hypotension.

What is the name of this complication of the transfusion?


Acute Haemolytic Transfusion Reaction

What is the most likely procedural reason for this complication to have arisen?
mismatching of donor blood group to that of the patients/ ABO incompatibility

Name 2 immediate steps that need to be taken.


stop blood transfusion & inform the hospital Transfusion Laboratory
Take a post transfusion sample from patient & send to Transfusion Laboratory

List 3 treatments that the patient might then urgently require.


Keep IV line open with saline 0.9%
Give high flow oxygen & IV fluid support
Monitor urine output and if falls below 100 ml/hr provide furosemide 40-80mg IV infusion with
0.9% saline
Provide inotropic support - dopamine 2-5mcg/Kg/min
steroids
adrenaline IM
chlorpheniramine

What might you detect in the urine?


haemoglobin

List 3 further short term complications of blood transfusions.


Transfusion related ALI
Febrile non-haemolytic reaction
allergic & anaphylactic reaction

30

Question 26: Urology


A twenty-four year old woman is admitted with a history of acute pain in the left loin,
radiating to the groin. She has dysuria and rigors and a temperature which is
measured at 40.5 centigrade. On examination she is acutely tender in the left renal
angle, where palpation demonstrates an obvious renal mass.

What is the likely diagnosis?


pyelonephritis

Name one aspect of the initial management


analgesia/ paracetamol 1mg O PRN (<4mg daily)
Antibiotics
IV Fluids

Give 2 abdominal investigations which would help elucidate the cause of the mass.
Renal USS ? hydronephrosis
Non-contrast CT pyelogram method of choice for suspected stones
(IV/contrast CT not recommended in renal insufficiency)

What consequent action should be taken if these tests reveal hydronephrosis?


Removal of the obstruction and drainage of the urine
If Higher Obstruction:
Acute nephrostomy tube
Chronic ureteric stent or pyeloplasty
If Lower Obstruction:
Urinary / suprapubic catheter

Name 3 possible consequences of delaying treatment.


development of additional stones
Stagnant fluids risk of infection sepsis
Obstruction builds pressure feedback pressure to kidney chronic kidney
damage

Give 2 methods by which kidney stones may be managed.


Conservative analgesia (NSAIDs / Morphine), fluids, tamsulosin, nifedapine
Surgical shock wave lithotripsy, laser probes, ureteroscopic lithotripsy, open
lithotomy

31

Question 27: Gastrointestinal/ Hepatic


A 74 year old woman presents with severe pain in her left iliac fossa. She is febrile,
has a tachycardia and her white cell count is raised.

What systemic clinical syndrome is present as a result of the abdominal problem?


Systemic Inflammatory Response Syndrome (SIRS)
On examination there is peritonism localised to the left iliac fossa

List 3 important differential diagnoses


Acute Diverticulitis
Locally Perforated Sigmoid Carcinoma
Inflammatory Bowel Disease
Ischaemic Colitis
Leaking AAA
Sigmoid Volvulus
Pyelonephritis

List 4 differential components of your initial management for this patient


Assess need for resuscitation using ABCDE
Analgesia
Bowel Rest i.e. Nil by Mouth + IV fluids
IV broad spectrum antibiotics as per local guidelines

What single investigation could best clarify the diagnosis?


Abdominal CT with Contrast

The following morning her signs have progressed and she has generalised peritonitis.
At laparotomy the sigmoid colon is found to be the cause of her peritonitis. What
operative procedure is indicated?
Hartmans Procedure

List 2 features of the pathology of sigmoid diverticular disease.


False Diverticula i.e. herniation of bowel wall that lacks outer coat of Muscularis propria
Muscular Hypertrophy of sigmoid

32

Question 28: GU
A 19 year old female attends her GP with dysuria five days following unprotected
sexual intercourse. She is not on any form of contraception and this was a casual
contact as she is not in a steady relationship at the moment

Name 2 sexually transmitted infections this patient may have contracted which would
be consistent with these symptoms
chlamydia
gonorrhea
mycoplasma genitalium
trichomonas vaginalis
HSV1/2

List 2 additional relevant questions you would wish to ask the patient.
Has she noticed any discharge? (thin and watery or thick/purulent STI)
Lower UTI Questions i.e. urgency, frequency, cloudy urine/offensive smelling Urine?
Upper UTI Questions i.e. loin pain, fever, chills?
Any blood in the urine?
Use of topical hygiene products e.g. scented soaps, vaginal sprays etc?
Any dyspareunia?
Any systemic symptoms e.g. fever?
Any PMH of UTI?
Any post coital-bleed?
If relevant, inter-menstrual bleed?

List 2 relevant examinations you would wish to undertake in this patient . .


Speculum Examination
Bimanual Palpation of Vagina

List 4 relevant investigations you might undertake


Urine Dipstick may show nitriles, leukocytes (UTI) +/- blood, protein
Mid-stream Urine Microscopy/Culture >105 CFUs = UTI
Urethral Culture + PCR for STDs e.g. HSV, Syphilis, HIV
Vaginal (high vaginal swab) Culture + PCR for STDs
Cervical (endocervical) Cultures + PCR for STDs
-HCG untreated UTI can cause premature/low birth-weight baby so vital to
know
WCC and CRP

Assuming she has a sexually transmitted infection, list 4 factors that would be
important at the next stage of management.
Compliance ensure antibiotics were taken for appropriate course
Test-of-Cure repeat diagnostic methods to confirm the infection has cleared
Education on safe sex and the perils of unprotected sex with strangers e.g. HPV
+ Cervical Ca risk
Contact Tracing if possible trace the original source and treat to prevent further
transmission
Contraception whether she has any/ if not pregnancy possibility / ? desire for
some
33

Advice avoid sex until test-of-cure proves infection is gone (prevents accidental spread)

34

Question 29: Respiratory


A 27 year old lady has returned from a holiday in Australia. She has been admitted to
hospital with severe left-sided pleuritic chest pain and a haemoptysis. She complains
of shortness of breath is cyanosed and tachycardic.

What would be the most important radiological investigation to carry out in the
Emergency Department and why?
CXR - exclude tension pneumothorax
Her arterial blood gases are as follows:
pO2 7.2 (normal range 10.5-14)
pCO2 3.0 (normal range 4.7-6)
Ph 7.39 (normal range (7.37-7.42)
Bicarbonate 20

What 4 investigations would you perform at this stage to help you with your further
elucidation of this patients problems and why?
D-dimer - if negative excludes likelihood of VTE
Doppler Ultrasound Lower Limb - exclude VTE
Lung Function Tests - indicate presence of obstructive/restrictive disease
FBC + CRP - CRP + WCC indicate infectious process

Name 3 forms of treatment that you would prescribe while you are waiting for the
results to come back
analgesia/ morphine - for chest pain
Flow (4l/min) concentration (28%) oxygen therapy via nasal mask
IV Fluids eg 250 mL saline 0.9%

What would the most important diagnosis be to exclude in this ladys instance?
PE

Which investigation would you use to confirm this diagnosis and what would it show?
CTPA - occlusion on pulmonary vasculature +/- distally threadlike vessels

35

Question 30: Geriatrics


An 82-year old lady is admitted to the Emergency Department after recurrent falls at
home. She has attended your clinic with a previous history of hypothyroidism,
Parkinsons Disease and hypertension. Her drug therapy includes Madopar,
Thyroxine, Bendrofluazide and Nitrazepam at night. On examination there are no acute
neurological findings, her pulse is 80 beats/minute and regular, blood pressure is
136/82 in a lying position and 102/60 when standing.

List 3 possible causes of a fall in this ladys case.


polypharmacy - sedatory effects of interactions of multiple medications
postural hypotension
cerebellar dysfunction due to Parkinsons

List 3 changes you would make to her drug therapy and explain why.
Bendrofluazide - consider lowering dose or changing according to national protocol to
calcium channel blocker
Nitrazepam - discuss the clinical need

Give 2 treatments that the physiotherapists might provide to reduce her risk of fall.
strengthening exercises therapy
balance exercises

What assessment might an Occupational Therapist carry out for this patient to prevent
further falls?
home assessment
need for social services

36

Question 31: Urology


You are working as a FY1 on a surgical ward when a 50 year old man, who has
previously enjoyed good general health, presents to outpatients with a clear history of
painless haematuria, without frequency or dysuria. Abdominal examination reveals
bilateral palpable kidneys and a marginally elevated blood pressure. You suspect he
has polycystic disease of the kidneys.

What other causes of painless haematuria are important to exclude in this instance?
List 4
prostatic cancer
benign prostate hyperplasia
renal stone disease
bladder tumours

What initial investigations would you like to perform at this stage that would help most
with the diagnosis? List 3
AXR - exclude any abdominal masses to indicate tumors
IV pyelogram - exclude obstruction due to renal stone disease
renal USS - determine presence of polycystic kidney disease
You find bilateral polycystic kidney disease without any other abnormality of the lower
urinary tract. You are concerned about the possibility of renal impairment and wish to
assess the level of renal function more accurately.

What tests are available to do this? List 2.


U&Es - urea, electrolyte disturbances indicate impaired kidney function
eGFR measurement - determine AKI

Name 2 possible alternatives available for long-term renal replacement therapy.


Haemodialysis/ Peritoneal dialysis
transplantation

List 2 factors which you would like to discuss with him before considering these
options.
risk & complications associated with each option
the impact any comorbidities eg disease may have on the safety/effectivity of transplant

37

Question 32: Infection


An 18 year old student presents with a 9 day history of persistent sore throat and
general malaise with an episodic fever. She has now developed a widespread
erythematous rash. Three days ago she started taking ampicillin capsules she had at
home, left over from a previous illness.

You suspect a diagnosis of infectious mononucleosis. List 2 abnormalities of a full


blood count that would support your diagnosis.
Lymphocytosis
mononucleosis cells (atypical activated T cells)
anaemia
thrombocytopenia

Give 4 clinical signs you may find on examination.


splenomegaly
petechial haemorrhages covering of soft palate
whitish exudate covering tonsils
lymphadenopathy
hepatomegaly
fever

What is the causative organism for this condition?


Epstein Barr Virus

What is the mode of transmission for this condition?


Salivary exchange eg kissing

What test would confirm your diagnosis?


Monospot Test

Give 4 complications of this disease.


Post-Viral Chronic Fatigue Syndrome
Splenic Rupture (esp if engaging in contact sports)
Guillain Barre Syndrome
encephalitis
meningitis
Cranial lesions
severe upper airway obstruction
hepatitis

What is the management of this condition?


supportive - analgesia, oral rehydration, stop antibiotics

38

Question 33: Gastrointestinal/ Hepatic


A 31 year old woman attends as an elective outpatient with a three-month history of
rectal bleeding and anal pain.

Is this patient likely or unlikely to have rectal cancer?


unlikely - <50yrs

After taking a full history, what assessment would you undertake in the clinic? (Name
3)
Digital Rectal Examination - assessing anal tone, presence of mass,
GI examination - assessing palpable masses, other causes
Fecal Occult Blood Test - presence of blood

After appropriate assessment, a diagnosis of haemorrhoids is made. What therapeutic


choices would you discuss with the patient? (Name 3)
topical lidocaine - symptomatic relief
rubber band ligation
injection sclerotherapy

What feature in the patients history would be the most important guide to choosing
an appropriate treatment?
whether there is any pain indicating likelihood of strangulation

Name one condition which may predispose to the development of haemorrhoids.


obesity

Name another common cause of rectal bleeding with anal pain in young adults.
fibre diet causing constipation leading to straining upon opening of bowels

39

Question 34: Infection


A 38 year old aid worker, while on the flight home from South America, has developed
cramping abdominal pain and profuse diarrhoea. You think he may have developed
cholera.

List 2 clinical features which may occur as a result of the profuse diarrhoea that is
caused by cholera.
dehydration - skin turgor, sunken orbits, postural hypotension, confusion,
weight loss, oliguria
electrolyte disturbance
shock

Given the history of travel, list 4 other organisms that should be looked for as possible
causes of the diarrhoeal illness.
Enterotoxic E. Coli
Salmonella
Campylobacter
Giardia
Entamoeba histolytica
Shigella
Cryptosporidium
Rotavirus
Norovirus
Plasmodium Falciparium

What organism causes cholera and how does the disease result following infection?
Bacteria - Vibrio Cholera (gram negative rods) -> clinical features due to enterotoxin release
It secretes an enterotoxin which stimulates adenylyl cyclase that the
concentration of cyclic AMP leading to persistent and excessive secretion of fluids
and electrolytes.

Intravenous fluid replacement is essential in cholera infections. Apart from the water
replacement, name the 2 most important constituents in the IV replacement fluid that
will help correct the sequelae of the profuse diarrhoea.
Sodium & Glucose
Pali suggest HCO3- and K but the Exemplar answers are the ones used above.

What advice should be given to travellers to avoid diarrhoeal illness?


CDC 5 basic cholera prevention messages:
Drink and use safe water (i.e. sealed bottle water)
Wash hands often with soap and safe water
Use latrines or bury your faeces; do not defecate in any body of water
Cook food well ("Boil it, cook it, peel it, or forget it.)
Clean up safely in the kitchen
Note: A vaccine for cholera is available; however, it confers only brief and incomplete
immunity and is not recommended for travellers

40

Question 35: Cardiovascular


An 84 year old woman with well controlled hypertension is admitted after several falls
at home. Her daughter reports that she looks pale, then falls and briefly loses
consciousness. There is no focal weakness or abnormal movements. She makes a full
recovery each time.

Apart from orthostatic (postural) hypotension, suggest 3 other disorders which are
likely causes for these symptoms.
vasovagal syncope
Arrhythmias e.g. Adam-Stokes Attack (complete heart block), sinus arrest or non-sustained
VT

On examination you find that her blood pressure drops markedly on standing up and
she feels faint. Suggest 2 factors which should be considered that might be
aggravating this change.
pharmacological therapy she is receiving
antihypertensive medication
over-diuresis
age

List 2 non-drug measures which may help to control postural hypotension.


Avoiding rapid postural changes i.e. stand up slowly
Staying well hydrated
Elevating the head of the bed while sleeping (reduces Nocturia)

What types of drug therapy would you consider prescribing in an effort to alleviate her
symptoms? Suggest 1.
Synthetic Corticosteroid with Mineralocorticoid action e.g. Fludrocortisone
0.2mg TDS for 1 week

The patient improves on treatment and you plan discharge. What other factors will you
consider in your plans? Suggest 2.
medication review
physiotherapy
OT assessment
dietetic assessment - food & fluids

41

Question 36: Respiratory


A thin, previously healthy 24 year old man presents to the Accident and Emergency
department complaining of sudden onset of left sided pleuritic chest pain and
breathlessness. You suspect a spontaneous pneumothorax.

Name 3 clinical signs would you expect to find on respiratory system examination.
resonance
decreased breath sounds
expansion
no vocal resonance/ tactile fremitus

Name 2 groups of patients who are at increased risk of developing spontaneous


pneumothorax?
young, thin, tall males
smokers
patients affected by Marfans Syndrome
FHx of pneumothorax
patients with underlying respiratory disease eg asthma, COPD, CF, TB

Name 2 factors that need to be taken into account when considering treatment of
spontaneous pneumothorax.
Severity of symptoms
Size of the pneumothorax i.e. <2cm or >2cm rim between lung and chest wall (estimated via
CXR)
Whether pre-existing lung disease exists or not (if so then lower threshold for drainage)
Over the next hour your patient becomes increasingly breathless and distressed.

What complication do you need to consider?


tension pneumothorax

Name 2 physical signs that you might associate with this problem.
tracheal deviation
cardiovascular instability
worsening hypoxia

In considering the patient for a chest drain insertion, you wish to avoid the costal
blood vessels. Where are these located and how can they be avoided?
Location costal grooves on the inferior surface of the ribs
How to Avoid insert the chest drain directly above the rib (4-6th on mid-axillary
line) aiming inferiorly

42

Question 37: GU
A 24 year old professional cyclist complains of painless left testicular swelling. You
suspect he has a testicular tumour.

List 2 other differential diagnoses of painful/painless testicular swelling.


testicular torsion
epididymo-orchitis
haematoma
granulomatous orchitis
TB

Name 2 other causes of painless scrotal swelling.


hydrocele
epididymal cyst
variocele
inguinal hernia

What would be the most standard radiological investigation to help with the
diagnosis?
Scrotal USS

Name the 2 common histological types of testicular tumour.


seminomas
teratomas

List 2 tumour markers that may be raised.


beta human chorionic gonadotrophin
alpha fetoprotein

Name 3 treatment options that are available for treating testicular tumour.
radical orchidectomy +/- testicular prosthesis/ sperm storage
chemotherapy if metastases eg pleomycin
radiotherapy (external beam)

43

Question 38: Gastrointestinal/ Hepatic


A 23 year old woman comes to see you in your general practice. She tells you she has
frequent episodes of diarrhoea. Initial examination reveals that she is short and thin.
Further questioning discloses that her bowel motions are often pale coloured and foul
smelling. This has been a recurring problem most of her adult life and you suspect
she may have coeliac disease

List 3 conditions that you would include in your differential diagnosis.


chronic pancreatitis
Inflammatory Bowel Disease (Crohns)
Lactose Intolerance
IgA deficiency
Giardia Infection
Cystic Fibrosis
Further history reveals no tendency to chest infections. There is no family history of
note. Physical examination is normal apart from her small stature and thinness.

List 2 investigations that will help to refine your diagnosis.


Bloods: FBC (Anaemia from malabsorption), ESR/CRP to exclude inflammatory cause
Serology: IgA, tissue transglutaminase Ab, IgA endomysial Ab
Stool Test: Sudan stain to check for steatorrhea
Peripheral Blood Film: Microcytic Hypochromic (Fe deficiency) due to reduced iron
absorption
You refer her to a gastroenterologist, who carries out an endoscopy and duodenal
biopsy. The biopsy report comes back as suggestive of coeliac disease.

List 2 pathological features that might have been seen on the biopsy.
subtotal villous atrophy
crypt hyperplasia

Are typical biopsy appearances diagnostic for coeliac disease?


yes assuming antibody tests are +ve and patient is not on a self-mediated gluten free diet

How would you suggest this patient should be treated and what investigation would
you suggest after treatment?
patient education to avoid gluten diet
repeat upper GI endoscopy biopsy post-6wks gluten absent diet which should reveal
resolution of initial pathological findings

44

Question 39: Haematology


A 32 year old woman, who is a mother of four children under aged 6 years, presents
with increasing fatigue and shortness of breath over recent months. She has no
significant past medical history.
You find her to be pale with: Hb 6.9 g/dl (Reference range 11-13 g/dl), MCV 63 fl
(Reference range 78-96 fl), MCH 24 pg (Reference range 27-32 pg)

What is the name given to this blood picture?


Hypochromic microcytic anaemia

What is the most likely haematological disorder in this lady?


iron deficiency hb

Name 2 possible significant factors underlying in this patient.


stress - mother of four young children, insufficient diet
multiple pregnancies

What arterial pO2 result would you expect?


75-100mmHg - normal as O2 content falls in proportion to haemoglobin

You examine the blood report to see if it provides a reticulocyte count. What are
reticulocytes?
immature enucleated (contain rRNA) erythrocytes - formed in Bone marrow

The reticulocyte count is normal. What is the significance of this finding?


It indicates the anaemia is likely due to dysfunctional erythropoiesis and not haemolytic in
origin
On further questioning you learn that her ethnic background is South Asian.

What co-existing blood condition may this patient have? How would you test for this?
a-thalassaemia
PCR assay testing/ haemoglobin electrophoresis

45

Question 40: Orthopaedics/ MSK


A 74 year old woman who was admitted to hospital via A&E having been found on the
floor at home. She is unable to get up and complains of pain and weakness in the right
leg. You suspect a fractured neck of femur.

What 2 features would you look for on inspection of the legs to confirm your
diagnosis?
Right/ affected side is shortened & externally rotated in comparison to the left

The x-ray demonstrates an intra-articular fracture of the right hip. What is the
anatomical significance of a fracture at this site?
involvement of the joint space, disruption of articular cartilage and smooth articular bone
surface
Blood supply to the femoral head is in a distal proximal fashion.
Site of intra-capsular fracture means that blood supply to femoral head is potentially
compromised and there is a risk of AVN to the femoral head

You are concerned about the diagnosis of osteoporosis in a lady with a previous
fracture. You proceed to undertake a DEXA scan. Which areas are routinely screened
for the presence of osteoporosis?
L1-L4 Lumbar Spine, femoral neck

Briefly (< 30 words) describe how the DEXA scan works.


two x ray beams with different energy levels are aimed at the patient's bones
soft tissue absorption is subtracted out, the BMD can be determined from the absorption of
each beam by bone.
BMD = total XR absorbed XR absorbed by soft tissue

Give 2 drug therapies which may be of benefit in osteoporosis.


alendronic acid (bisphosphonate)
calcitriol

Given the above information, what operation would be most appropriate?


replacement hemi-arthroplasty

She makes a good recovery from the operation. Name 2 factors related to her social
situation that you would wish to take into account when planning her discharge.
home assessment and suitability of home environment considering her mobility
status
need for social services to help with daily activities
She lives alone who will look after her?
What is her home like hazards, stairs, where is the bathroom, shower?

46

Question 41: Urology


A 36 year old female presents to her GP with nocturia, nausea and generalised pruritis.
She has a history of enuresis and urinary infection in childhood and five year history
of hypertension. There is no other past history of note. The GP finds her BP 150/90
and serum creatinine to be 1200mol/l (reference range 58- 124mol/l). The patient is
then referred to the medical unit where you are working as a FY1.
You request a number of investigations (blood and imaging).

Give 4 investigations that would help you to determine whether her renal failure was
chronic rather than acute.
USS to see size of the kidney
FBC to look for anaemia normochromic anaemia
Parathyroid Hormone seen in Renal Osteodystrophy
X-ray chondrocalcinosis at knees/pubic symphysis; osteopenia and bone
fractures look for renal osteodystrophy

Whilst awaiting the results of investigations you consider that dialysis may be
required. Give 4 clinical features and/or laboratory findings that would help guide your
decision.
Diuretic resistant pulmonary oedema
Hyperkalaemia (refractory to medical therapy)
Metabolic Acidosis (refractory to medical therapy)
Uraemic Complications (pericarditis, encephalopathy, bleeding)
Dialysable Intoxications (eg, lithium, toxic alcohols, and salicylates).

Your investigations suggest chronic renal failure. What 3 forms of renal replacement
therapy are available?
Haemodialysis
Peritoneal dialysis
Transplant
Other - continuous renal replacement therapies (CRRT).

Which is the best form of renal replacement therapy in the long term?
live-related renal transplant

47

Question 42: Neurology


You are on a GP attachment when a 24-year old shop assistant comes to see you with
a 3-day history of pain around the right eye, associated with reduced vision in that
eye. She is unable to see small objects or distinguish colours. You find that her visual
acuity is 6/36 in the right eye and 6/6 in the left. The right optic disc is swollen and
there is a right afferent pupillary defect. You see in her notes that one year previously
she had experienced an episode of numbness affecting both legs and associated with
urgency of micturition and temporary urinary incontinence. This had all resolved
spontaneously and had been attributed to a trapped nerve. You suspect a clinical
diagnosis of multiple sclerosis.

What is the appearance seen in the eye?


swollen optic disc

What is an afferent pupillary defect?


dysfunction of pupil musculature impacting ability to constrict or dilate iris normally
In a Marcus Gunn pupil, there is reduced afferent input and the pupils fail to constrict fully.
Stimulation of the normal eye produces full constriction in both pupils. Immediate subsequent
stimulus of the affected eye produces an apparent dilation in both pupils since the stimulus
carried through that optic nerve is weaker.

What is the immediate cause of the visual loss?


inability to conduct efferent signals from the optic n. to optic lobe of cerebral cortex due to
neuropathy - optic neuritis

Suggest 2 investigations that should be undertaken and what results you will expect
from them?
LP + CSF analysis - detection of intrathecal inflammation, pleocytosis, protein,
oligoclonal bands
brain & spinal cord MRI with gadolinium labelling - presence of plaques in cortex, brainstem,
spinal cord

48

The further investigations confirm the diagnosis, which has already been guessed by
the patient. She is now well but comes to see you to talk things over. What points
will you want to make with regard to prognosis and management? Suggest 4.
It is usual practise to inform the patient that there is no cure for MS
There is no method for predicting the course of MS and there is wide variation in its severity
Many MS patients live self-sufficient lives, while others are gravely disabled
MS team and specialised services for one-to-one counselling, support, education and
information
Treatment:
For Relapses Corticosteroids and Immunosuppressants
For Symptomatic Relief Muscle Relaxants (for spasticity) and Anticholinergics
(for tremor)
To Frequency of Relapses (e.g. INF-) but they only work in select group of
patients and are limited in their efficacy

What aetiological factors are associated with this condition? Suggest 2.


Genetic HLA-A3, IL-7RA, IL2RA, Having First/Second-degree Relatives with MS,
Being
Environmental Living further from the equator, Smoking, Vitamin D Deficiency,
EBV Infection,

49

Question 43: Emergency Medicine


20-year old university student is brought into the A & E Department by a friend having
taken an overdose of paracetamol (approximately 20 tablets about 5 hours ago). She is
fully conscious and cooperative but withdrawn and admits to feeling depressed
recently

What is the most important investigation to undertake at this stage and why?
Plasma Paracetamol Levels - Concentration will guide management

What would you administer orally to reduce the absorption of the paracetamol?
Activated Charcoal

List 3 other important investigations that would also be undertaken and why.
Coag Screen hepatotoxicity of paracetamol may impair liver synthesis of
clotting factors
Urea and Electrolytes assess renal damage and electrolyte levels/ for baseline
measures
Arterial Blood Gas paracetamol overdose can cause acidosis
LFTs same reason as coag

What is the most important treatment of paracetamol overdose at this stage and how
does it work?
N-Acetyl-Cysteine (Parvolex) - Glutathione Donor and therefore serves to help bind toxic
metabolites

Over the next three days she becomes mildly jaundiced. Then, over the subsequent 48
hours she becomes increasingly jaundiced, agitated and confused. What is the likely
diagnosis and what action can be taken?
Hepatic Encephalopathy due to Liver Failure
Refer to tertiary centre for liver transplant

50

Question 44: Gastrointestinal/ Hepatic


You are on a General Practice attachment. A 57 year old man, who is an infrequent
attender but who has smoked approximately 20 cigarettes per day for the last 30
years, presents with an 8-week history of difficulty in swallowing. He has been
previously well until his symptoms developed. Since then the problem has become
progressively worse. The patient denies any symptoms of heartburn. On direct
questioning he has lost 5kg in weight over this time period.

Apart from carcinoma of the oesophagus, list 4 recognised conditions which could
cause dysphagia in any patient.
diffuse oesophageal spasm
achalasia
food bolus/ foreign body
benign oesophageal stricture
external compression from enlarged thyroid
GORD -> oesophagitis -> oesophageal stricture
gastric cancer
CVA/ stroke
pharyngeal pouch/ web/ diverticulum
carcinoma of bronchus

Give 2 features in any patients history which would suggest a malignant cause of the
dysphagia.
Progressive dysphagia
Weight loss
no PMHx of reflux Sx
rapid onset/ short history
PMH Smoking NOT the best answer since the question is asking for features
not risk factors

You decide that the patients symptoms merit investigation. What 2 key investigations
could you request?
OGD +/- Biopsy of any abnormal tissue
Ba Swallow (only way of diagnosing functional/motility problems)

Name the 2 main pathological types of oesophageal carcinoma.


adenocarcinoma
squamous carcinoma

51

The referring consultant considers a surgical resection for this gentlemans


oesophageal carcinoma. What investigation is now important and why would you
undertake it?
CT brain chest abdo pelvis :
- stage the disease
- show tumors vol
- presence of metastases
- to assess appropriateness of resection.
If not invaded beyond submucosa then surgery alone is indicated since there is no evidence
of additional significant benefit from concurrent chemoradiotherapy. However, if any local
spread beyond this then chemoradiotherapy is indicated, while the presence of any
metastases would preclude excision. Another key investigation is Endoluminal US to assess
local invasion.

Surgical resection proves impossible. What 2 courses of action will help to palliate his
symptoms?
endoscopic laser surgery for lesions <8 cm long
oesophageal stenting with a Celestin tube if longer than 8cm
Radiotherapy either external beam or brachytherapy to alleviate dysphagia
Endoscopic Ablation +/- Stenting
Opiates pain relief
Chemotherapy alone is not good palliation

52

Question 45: Gastrointestinal/ Hepatic


You are a FY1 attending a general surgical outpatient clinic. A 60 year old man
presents with a six month history of intermittent rectal bleeding.

Apart from rectal carcinoma suggest 3 other likely causes for these symptoms.
Anatomical Diverticular Disease, Meckels Diverticulum (more common to present
in children)
Vascular Colonic Angiodysplasia
Infection Infective Colitis e.g. Campylobacter, Salmonella, Shigella
Anorectal Haemorrhoids, Anal Fissures
Inflammatory BD with blood more likely UC though both would typically present @
<50yrs

Name 3 further features in the history you would seek to support a diagnosis of rectal
carcinoma.
Change in Bowel Habit frequency loose stools +/- mucus passed
Tenesmus
Weight Loss
Family History of Colo-rectal cancer (?FAP/ HNPCC)
Past Medical History Inflammatory BD, Polyps or Colorectal Cancer
Awareness of Mass

Further investigations lead to the resection of a rectal carcinoma. Prior to discharge


the patient asks you about his prognosis. What important factors do you need to take
into account in order to determine prognosis? Suggest 3.
DUKEs:
Local Spread i.e. whether it has penetrated the bowel wall or not (yes ~66% 5yr survival)
Lymphatic Spread i.e. whether there is lymph node involvement (yes ~33% 5yr survival)
Distant Metastases i.e. whether other structures are involved (yes ~5% 5-yr
survival)
+/-: - Success of surgery i.e. if it was completely or partially resected, - Co-morbidities

Name 2 medical conditions which may predispose patients to rectal carcinoma.


ulcerative colitis/ crohns disease
adenomatous polyps
hereditary polyposis
HNPCC (Lynch Syndrome)

53

Question 46: Cardiovascular


A 39 year old Asian man was admitted to the medical admissions unit with pains in his
chest and neck. He admitted to smoking 20 cigarettes per day, and a blood cholesterol
had been measured at 7.2mmol/l (reference range 3.5 5.0 mmol/l). His average heart
rate on admission was 90 beats per minute and his blood pressure was 170/100
mmHg. An initial diagnosis of unstable angina was made.

What are his risk factors for coronary artery disease? List 4.
smoking - 20 cigarettes per day
Asian ethnicity
hyperlipidemia/hypercholesterolemia (7.2mmol/L cholesterol)
Hypertension (170/100mmHg)
male

You decide to admit him to hospital. What drug therapy could he be started on? List 4
potentially beneficial drugs (2 marks) and give a reason for prescribing each (2
marks).
Simvastatin 5-50mg nocte O - cholesterol & mortality
Calcium Channel Blocker/amlodipine - hypertension by vasodilation & controls
angina
Nitrates/ Glyceryl Trinitrate Spray PRN - symptomatic relief of angina, BP by
vasodilation
Metoprolol 100mg BD - vasodilate coronary arteries to maintain perfusion and
ischemic episodes, BP, HR, controls angina
Aspirin 300mg O bolus ->70mg O OD - prevent platelet aggregation & activation
Clopidogrel/ ticagrelor 90mg O BD - alternative to aspirin and reduces mortality
Lisinopril 5mg BD -> 5-10mg OD- BP & mortality
morphine - controls pain and helps patients feel at ease
Results of blood tests revealed a Troponin T of 0.35ng/ml. (N: unrecordable), peak
Creatinine kinase was 180 iu/ml (reference range: 25- 200 iu/ml) on day two.

List the 2 cardinal ECG features of an acute full thickness anterior myocardial
infarction and outline their electrophysiological cause.
ST elevation (V1-6) +/- (V1 & aVL) - changes in action potentials produced by necrotic
tissues, abnormal firing of action potentials leads to early repolarisation secondary to
ischemia causing this abnormal wave
Pathological Q waves (V1-6) +/- (V1 & aVL) - develop from living tissue behind the infarct,
picked up by ECG as downward movement as impulses move away from anterior leads
Reciprocal ST depression in inferior leads (VIII-aVF)

54

Question 47: Gastrointestinal/ Hepatic


A 48 year old gentleman presents to his GP with a 6 month history of epigastric pain
which is made worse by eating.

Name 3 organs which may cause food related pain.


stomach
pancreas
gallbladder

On examination there is mild tenderness in the epigastrium. Your consultant tells you
that the Murphy's sign is negative. What is the significance of a positive Murphy's
sign?
Positive arrest of inspiration on palpation in the upper right quadrant but NOT
in left upper quadrant
Positive result due to pressure (from palpation) on peritoneal inflammation 2 to an inflamed
gallbladder
Hence, it suggests acute cholecystitis is the likely cause of his pain

You suspect the patient may have a peptic ulcer. Name 1 investigation which could be
performed to confirm the presence of Helicobacter pylori?
Urease Breath Test
H. Pylori Stool Antigen Test
The above 2 are correct for a patient presenting to a GP. If the Hx was presenting
to ER/Surgery OGD + CLO test and biopsy

The patient undergoes endoscopy which reveals an ulcer in the lesser body of the
stomach and triple therapy is commenced. Briefly outline the action of lansoprazole in
reducing acid secretion.
Lansoprazole is a Proton Pump Inhibitor (PPI)
PPIs irreversibly block hydrogen/potassium ATPase in parietal cells of the stomach
The result is massively reduced H+ secretion stomach acid formed

Two weeks later the patient presents with a rigid, tender abdomen, highly suggestive
of perforation. Name 2 other complications of peptic ulceration.
Acute Upper GI Bleed
Fe Deficiency Anaemia 2 to chronic blood loss
Gastric Outlet Obstruction (long-term complication)
Gastric Cancer (long-term complication)
Penetration

What important initial investigation should now be performed if you suspected


perforation and what would it show?
Erect CXR air under the diaphragm i.e. pneumoperitoneum

55

Question 48: Haematology


You are a GP trainee and Mrs J, a 24 year old lady, presents to the clinic. She has just
registered at the practice, her notes havent arrived at the surgery and she is
complaining of tiredness. She doesnt wish to be fully examined but clinically you
suspect anaemia and you arrange a full blood count, results of which are shown
below.

She returns to the health centre and this time you notice that she is also slightly
icteric. Urine analysis shows urobilinogen but no bilirubin. There is no glycosuria,
haematuria or pyuria. The serum bilirubin concentration is 65 mols/l (normal range 15
22 mols/litre).

Apart from investigations for haemolysis, list 2 other investigations, explaining your
reason for doing the test, to help elucidate the cause of the increased MCV.
B12 deficiency can cause Macrocytic Megaloblastic Anaemia
Folate Levels deficiency can cause Macrocytic Megaloblastic anaemia
TFTs hypothyroidism can be a cause of macrocytic anaemia
Serum Protein Electrophoresis check for paraproteinaemia (myeloma)
Bone Marrow Aspirate/Trephine check for myelodysplastic syndrome

Apart from results given above, list 2 biochemical or haematological abnormalities


that may occur in haemolysis.
Biochemistry high LDH, high unconjugated serum bilirubin, haemoglobinuria,
haemosiderinuria
Haematology increased reticulocyte count, methaemoglobinaemia

Explain (in less than 50 words) why in haemolysis increased serum bilirubin may not
lead to increased renal excretion of bilirubin.
Haemolysis results in an increased number of red blood cells being broken down
The above causes an increase in the amount of unconjugated bilirubin in the blood
Unconjugated bilirubin is not soluble in water and hence not excreted by the kidneys

Apart from haemoglobinopathies, list 1 defect in the red cells that can cause
haemolysis and give an example.
Abnormal Membrane e.g. Hereditary Spherocytosis or Elliptocytosis
Abnormal Enzymes e.g. G6PD deficiency, Pyruvate Kinase Deficiency
Abnormal Haemoglobin e.g. Hb C, Hb S, Unstable Haemoglobin
It transpires, when the notes arrive, that Mrs J had a splenectomy for this problem as
a child and that she has subsequently had no follow up or treatment after this
procedure.

List an organism you would wish to vaccinate against.


56

Strep. Pneumoniae
Haemophillus Influenzae
Neisseria Meningitidis
List 2 other pieces of advice you would wish to investigate in her instance.
Lifelong Prophylactic Antibiotics
Annual Influenza Vaccine
Pneumococcal Vaccine every 5yrs
If Relevant Anti-Malarial Precautions

57

Question 49: Anatomy


A 68 year old woman presents with an enlarging swelling in the area of the right groin.

What 3 structures, other than hernia, might give rise to a lump in this area?
subcutaneous fat (lipoma)
femoral lymph nodes (lymphadenopathy)
psoas m. (abscess)
femoral a. (aneurysm)
After your examination you are sure that you are dealing with a hernia.

What factors would influence your advice to the patient about the possibility of
surgical repair? Name 4
location of hernia - distinguish whether femoral/ inguinal
whether it is reducible/ strangulated?
presence of symptoms (eg pain)?
pts desire for surgery/ functional impact
pts comorbidities/ contraindications to surgery

What are the symptoms of obstruction in a groin hernia ?


pain
constipation
tender/ distended abdomen
nausea/ vomiting

58

Question 50: Respiratory


You are working in General Practice when Mr TJ, a 70 year old man, attends your
surgery. He complains that his cough is now giving him some problems. He has had a
cough on and off for the past 5 years, but he has recently noticed that he is having
difficulty getting up stairs and he is producing more phlegm than usual. He has
smoked 20 cigarettes per day for approximately 50 years. You arrange for Mr TJ to
undergo baseline spirometry. The results are reported as follows:

List 2 parameters, in the normal population, that predict lung function in nonsmoking
subjects.
age
height

What pattern is demonstrated by the spirometry in this case?


obstructive (FEV1 <80% predicted + FEV1:FVC <0.7)

List 2 further noteworthy features indicated by the results.


Reversibility testing is negative indicating that the obstruction is irreversible
TLC has increased indicating lung hyperinflation
RV/TLC is greatly increased confirming the obstructive pattern
DLCO is decreased indicating active alveolar surface area has been reduced due to e.g.
Emphysema

You also arrange for some baseline blood tests, which show a raised haemoglobin of
19.8g/dl. What is the physiological explanation for the raised haemoglobin?
Gaseous exchange is impaired
As a result, less oxygen is readily absorbed into the bloodstream
Resulting hypoxia is detected by kidneys (juxtaglomerular app.)
erythropoietin production Hb
You discuss with Mr TJ the need to stop smoking and provide him with a number of
medical reasons to encourage him to stop. You review Mr TJ four weeks later, only to
be informed that he has not been able to reduce his cigarette consumption.

59

List 3 further methods of smoking cessation, which you discuss with Mr TJ


Community smoking cessation group
Nicotine replacement therapy (patches, gum, lozenges, sprays, inhalers)
Champix (Varenicline) therapy
OR
Biological nicotine replacement, bupropion, Champix (Varenicline)
Psycho-Social smoking cessation groups, hypnosis, counselling, monetary benefits,
quit4u scheme

60

Question 51: Endocrinology


A 31 year old woman attends her GP practice with a history of weight loss,
palpitations, sweating and shakiness. She has been previously well and had no
significant childhood problems. On examination she might have a goitre and has a
rapid pulse at rest (110 beats/min).

Apart from thyroid disease, which other common condition do you wish to exclude?
anxiety
Can mimic the increased heart rate and agitation of hyperthyroidism but palms will be
clammy instead of warm. Other signs may include the existence of goitre, eye signs,
proximal myopathy and wasting.

The patients Thyroid Stimulating Hormone (TSH) is undetectable (<0.05mU/L). In view


of this, what diagnosis does this indicate?
Thyrotoxicosis most likely due to primary hyperthyroidism

Why is the TSH low (give an explanation in no more than 30 words)?


TSH is produced by the anterior pituitary, and stimulates production of thyroid hormones (T3
&T4)
Increased thyroxine inhibits the hypothalamic-pituitary-thyroid axis as part of a -ve feedback
loop
The above results is production of TSH

Other results include:


Thyroid peroxidase antibodies: 1000 iU/L (N: <60 iU/L).
White Cell count: normal
Alkaline Phosphatase: 200 iU/L (N: 40-130 iU/L).

What is the likely specific aetiology of the disease?


Graves Disease autoimmune condition resulting in hyper-active thyroid

In no more than 30 words, outline the pathogenesis of this disease.


Autoantibodies to the TSH receptor thyroid follicular epithelial cells are created
These autoantibodies stimulate inappropriate activation of thyroid hormone production

Outline 3 treatment options for this patient, and provide one specific side effect that
you would warn the patient about for each treatment option
Anti-thyroid Drugs e.g. Carbimazole or Propylthiouracil can cause
agranulocytosis
Radioactive Iodine may cause initial worsening in hyperthyroid symptoms
Thyroidectomy likely hypothyroidism needing lifelong thyroxine treatment
Propranolol exercise intolerance

61

Question 52: Endocrinology


A 54 year old man with longstanding inflammatory bowel disease presents with an
increase in bowel frequency, passage of mucus and blood per rectum and a two
month history of 4kg weight loss.

Give 3 possible explanations for the patients change in symptoms.


Exacerbation of inflammatory bowel disease
Infective Colitis
Colorectal Cancer
Ischaemic Colitis
Coeliac Disease
Chronic Diverticulitis

Name 4 non-invasive routine investigations you would undertake.


FBC (WCC in infection/ check Hb to assess severity of loss/chronic disease)
WCC (? inflammation)
ESR/cRP (? Inflammation)
U+Es (may be deranged if chronic diarrhoea)
Albumin (acute phase protein/ surrogate marker of nutrition)
Culture Stool and Blood (exclude infective causes)
Faecal Calprotectin (evidence of colonic inflammatory pathology)
AXR (exclude evidence of large bowel obstruction/ toxic megacolon)
CT Abdomen/Pelvis (will show colonic inflammatory change and may give indication about
mesenteric circulation)

What urgent investigations would you consider to obtain a biopsy sample?


Flexible Sigmoidoscopy (in practice colonoscopy in case pathology turned out to be
proximal)

Name the characteristic features seen on pathological examination of the biopsy


material.
Continuous Mucosal Inflammation (UC) or Skip Lesions (Crohns)
Crypt Abscesses
Transmural Inflammation
Ulceration/Fissuring
Granuloma (Crohns)

Biopsies confirm chronic ulcerative colitis with areas of severe dysplasia. Give 2
treatment options.
Panproctocolectomy with terminal ileostomy OR creation of pouch and ileoanal anastomosis
Subtotal colectomy with end ileostomy formation (emergency situation)

62

Question 53: Haematology


A 24 year old male attends medical outpatients with a four week history of unilateral
painless cervical lymphadenopathy. He denies any symptoms of a sore throat or upper
respiratory tract signs. He has lost 7kg in weight over 2 months and has suffered
drenching night sweats and generalised itch.

What is the single most likely diagnosis?


Hodgkins lymphoma

What other clinical findings might be present?


splenomegaly
symptoms of anaemia
c
hepatomegaly
?purpura, easy bruising

What initial laboratory investigations would you wish to perform?


FBC + differential - thrombocytopenia, pancytopenia
Blood smear - nucleated RBC, giant platelets
Excisional/ core LN biopsy - positive
LDH (prognostic) / ESR
HIV testing - excl HIV infection
(usually of little value) BM aspirate + trephine - cellular marrow with few large binucleate
cells having moderate cytoplasm and prominent nucleoli (Reed Sternberg cells) in a
polymorphous background

Name 2 causes of cervical lymphadenopathy in any adult patient, not including your
answer earlier
Reactive following viral infection (eg EBV, CMV, HIV)
seroconversion in HIV
Lung mets
Rheumatology - SLE, Juvenile chronic arthritis

63

Question 54: Cardiovascular


Mr R.T. is a 59 year old man who comes to see you in the Well Man clinic. He has no
symptoms and urine dipstick testing is negative. His blood pressure is 150 systolic, 98
phase 5 diastolic.

What is meant by phase 5?


the P at which the sound disappears

What will be your initial approach to this blood pressure result? Give two examples of
your next course of action.
measure it again in order to minimise the risk of white coat hypertension
offer ambulatory BP monitoring or home BP recording to dx HTN

You later decide to treat his raised blood pressure, but Mr R.T. is unhappy about long
term drug treatment and asks what benefits are likely. Suggest 2 long-term
advantages.
chance of having a stroke (HTN increase risk 2x)
chance of having a MI (HTN risk 3x)
risk of developing HF

What drug treatment will you consider? Suggest three classes of drugs likely to be
most useful.
ACEi
CCB
Thiazide like diuretics

Mr R.T. asks you about the side effects of the drugs that you propose to use. List one
potential adverse effect for each class of drug in your previous answer.
ACE-I cough, first-dose severe hypotension
Ca-Channel Blockers ankle swelling, headache
Diuretics hypokalaemia, postural hypotension, diabetes, gout

64

Question 55: Cardiovascular


An anxious 47-year old man who comes to see you when you are a House Officer
attached to a General Practice. He has recently stopped smoking and has a family
history of ischaemic heart disease. He describes intermittent anterior central chest
pain and is worried that he may have angina.

What two other common causes of such pain will you want to consider?
Pericarditis/ Myocarditis
GORD
coronary a. spasm
atrial fibrillation
aortic stenosis/ aortic regurgitation

You are unable to make a diagnosis on the history and examination is normal. You
decide to refer him for tests. Suggest two tests giving the rationale for each.
Invasive coronary angiography - gold standard for assessing presence/ severity of CAD
Stress ECHO - assist diagnosis and provide information on precipitating causes (eg aortic
stenosis, HOCM etc)
Exercise Stress Testing - determine degree of functional impairment in CAD

Angina is diagnosed. What medication will you consider to treat his pain? Suggest
two groups of drugs
Short & long acting Nitrates - breakthrough & prophylaxis (eg GTN spray & isosorbide
dinitrate)
Beta blockers - frequency of attacks
Calcium Channel blockers
Nicorandil/ K channel agonist
Ranolazine
Ivabradine

What other additional treatment will you consider? Suggest two.


aspirin
statins
ACE inhibitor/ angiotensin receptor blocker

Treatment is only partially successful and he is referred for coronary arteriography.


This shows partial blockages of both branches of the left coronary artery. What are
these two branches called?
L circumflex a.
L anterior descending (interventricular) artery

65

Question 56: Cardiovascular


Mr GH is a 78 year old man who sustained a large myocardial infarction 2 years ago.
Initially he seemed to recover well but recently has become more short of breath on
exertion and has developed swollen ankles This has failed to respond to your
treatment with Furosemide and Slow K and has necessitated his admission to
hospital. You suspect he may have developed heart failure.

What additional points could you seek in the history that would support this
diagnosis? Suggest 3.
Breathing Exertional Dyspnoea, Orthopnoea, Paroxysmal Nocturnal Dyspnoea
Exercise Poor Exercise Tolerance, Fatigue and Weakness
Lungs Cardiac Wheeze, Nocturnal Cough with Frothy Pink Sputum
Other Impaired Mental Status, Cold Peripheries, Impaired Urine Output During
Day with Nocturia

What neurohumoral systems may be activated in heart failure? Give 2.


Sympathetic NS Activity: - Fall in CO detected by baroreceptors
catecholamines HR/BP
RAAS: - renal perfusion activates RAAS angiotensin II ( PVR) and aldosterone
(BV) BP
ADH: - Low BP and angiotensin II trigger release resorption BV BP
Natriuretic Peptides - ANP and BNP released in response to sympathetic
stimulation, atrial distension and AG II. Both inhibit RAAS so BV/BP making
them beneficial. However, they are not released in sufficient enough quantities to
make a significant difference

You arrange for a chest X-ray. What features would support your diagnosis? Suggest
3.
Alveolar Oedema
Kelsey B Lines/ interstitial odema
Cardiomegaly
Dilated Prominent Upper Lobe Vessels
Pleural Effusion

You later consider starting the patient on an ACE inhibitor. What precautions will you
take? Suggest 2.
Warn about side effects e.g. first-dose hypotension, cough, hyperkalaemia, renal impairment
Start at a low dose and titrate up
Check Urea and Electrolytes before initiating treatment (? hyperkalaemia or renal
dysfunction)
Check patient is not taking drugs that have interactions with ACE-I e.g. Ciclosporin ( risk of
K+

66

Question 57: Dermatology


Miss FA is a 16 year old Caucasian who has been seeing her General Practitioner
every 2-3 months since childhood, with occasional visits to the Dermatologist
complaining of widespread severe itching and dryness of the skin. The condition has
fluctuated in severity over the years but had never cleared completely.

You think that atopic eczema (atopic dermatitis) is the likely diagnosis. What two
further points in the history would you seek to support your diagnosis?
Family history of atopy (asthma, eczema, allergic rhinitis, hay fever)
Personal history of other atopic disease
Sleep disturbance due to itch
Effect of previous treatment on patient
Aggravating and relieving factors

What two clinical signs would you seek on examination of the skin to support your
diagnosis?
Lichen Simplex Chronicus (lichenification)
Hyperlinear Palms
Keratosis Pilaris
Flexural Distribution
Erythema, Vesicles, Exudates
Prurigo Nodularis (thickened firm nodules)

Give the 2 main approaches to the topical treatment of this disorder.


Emollients:
Yellow Soft Paraffin Brand names e.g. Dermol are NOT accepted answers
Liquid Paraffin
Topical steroids:
Mild Hydrocortisone Acetate
Moderate Eumovate
Potent Betomethasone
Very Potent Dermovate

What advice would you give to the patient about the natural history of her skin
disorder? Give 2 points.
Atopy; genetic predisposition due to hypersensitivity to certain allergens. Begins in early
childhood but tends to improve with age. Increased IgE antibodies cause an increased
immune response to environmental allergens. It is therefore very important to avoid exposure
to known allergens or irritants.
Reduced barrier function of skin; skin cells are less tightly packed and have a reduced
waterproof barrier (filaggrin mutations), this makes it more susceptible to external agents and
increases moisture loss causing irritation, allergies, infection and dryness of the skin. It is
therefore very important to use lots of moisturisers to replenish moisture and improve the
barrier function of the skin.

What occupational advice would you give to the patient? Suggest 2 facts
67

If the place of work exposes the patient to irritants or allergens it is important that the patient
takes necessary actions to eliminate exposure, as her eczema cannot improve until these
factors are removed.

68

Question 58: Gastrointestinal/ Hepatic


A 25 year old accountant is admitted as an emergency under your care with a oneweek history of fever, malaise with marked diarrhoea with blood through the stool.
When he is weighed he notices that he has lost 5 kg in weight but on questioning he
has no previous relevant history. Abdominal examination shows tenderness in the left
side of the abdomen and you suspect a diagnosis of ulcerative colitis.

Name three common causes of these symptoms (apart from ulcerative colitis)?
infective gastroenteritis
IBD
bleeding peptic ulcer

Apart from undertaking a colonoscopy and biopsy, list two ways in which you could
help clarify the diagnosis.
Bloods: FBC, U&E, ESR, CRP, LFT
Blood culture and stool culture exclude infective causes
Sigmoidoscopy and rectal biopsy
Colonoscopy
Barium enema

What abnormalities are you likely to see in the rectal biopsy once this is performed?
Suggest one
UC

List two classes of drugs that might be used to treat this condition and give one
example of each.
Steroids Hydrocortiosone, Prednisolone
5-Aminosalicylic Acid Sulfsalazine, Mesalazine, Olsalazine
Steroid Sparing Agent Azathioprine

The patient deteriorates despite the drug treatment and abdominal pain becomes more
of a feature with swelling. What acute complication would you be concerned about
and how would you investigate this?
Toxic megacolon +/- perforation

What urgent operation might be appropriate?


Subtotal colectomy with end ileostomy and over-sewing of the rectal stump

69

Question 59: Cardiovascular


You are on a General Practice attachment. A 73 year old lady comes to see you
because she has developed a 10cm2 ulcer above her left medial malleolus

Name 3 pieces of information you would enquire about in the history to support the
diagnosis of the ulcer being venous in origin.
Hx venous disease, DVTs or trauma (including surgery/fractures to leg) - 3 most
important points
Recurrent phlebitis
Previous pregnancy
Obesity
Immobility
History of pro-thrombotic tendency

What will you look for on examination to support the diagnosis of the ulcer being
venous in origin? Suggest 3 findings
Anatomical Location located around gaiter area
Shape typically a shallow ulcer with flat margins though not necessarily
Palpable peripheral pulses and normal CRT
Signs of venous hypertension i.e:
- Varicosities
- Haemosiderin Pigmentation
- Lipodermatosclerosis,
- Venous Eczema
- Atrophie Blanche (can be 5 separate points)

You decide to refer the patient to hospital for further assessment. List 2 investigations
that should be performed.
Ankle Brachial Pressure Index
Swabs exclude cellulitis (in practice nearly all ulcers will be infected to some
degree)
Blood Glucose
Biopsy - rarely done and of limited relevance

Venous ulceration seems likely. Suggest 2 curative measures that will be considered.
Multilayer-Compression Dressing
Cleansing and debridement
This is a bad question due to the word curative. Compression dressing is the better
answer here.

When the ulcer is healed, what advice will you give the patient to minimise the
chances of recurrence? List 2 pieces of advice.
Avoid prolonged standing/sitting and encourage walking with elevation at rest
Wearing correctly fitted compression stockings
Avoid injuring legs and maintain good skin care
Weight loss
Smoking cessation
70

Question 60: Neurology


A 21 year old man has been in a fight outside a pub. He has received at least one blow
to the left side of his head from a baseball bat. He localises and opens his eyes to
pain. He has incoherent speech.

In no more than 30 words, describe how you calculate his Glasgow Coma Score (GCS)
Eyes: - 2/4 i.e. open to pain
Verbal: - 2/5 i.e. incomprehensible sounds
Motor: - 5/6 i.e. localises to painful stimuli

What are the results in this case?


Total Score = 9/15

Is he in a coma?
No as Coma is a score <3

What urgent investigation does he need?


Head CT

What precaution needs to be taken first?


secure airway

In the context of an intracerebral injury, what is coning?


Coning refers to a tonsillar herniation i.e. when raised ICP causes the cerebellar tonsils to
move downward through the foramen magnum and press on the brainstem causing
potentially fatal respiratory and cardiac dysfunction.
You suspect an extradural haematoma (SDH).

Describe 3 signs of an expanding EDH as it enlarges and before it ultimately results in


coning.
Decreasing level of consciousness ( GCS)
CN III Palsy i.e. non-reacting dilated pupil looking down and out
Left Papilloedema
Increasing agitation
Worsening Headache
Nausea and Vomiting
Focal Neurological Signs

71

Question 61: Rheumatology


A 50 year old woman has had systemic lupus erythematosus (SLE) for 12 years. She
has developed ankle oedema and has proteinuria on urinalysis. You suspect she has
developed nephrotic syndrome.

What 2 laboratory results would confirm the clinical diagnosis of nephrotic syndrome?
(Give figures)
hypoalbuminuria (<30g/L)
proteinuria (>3.5g/24hrs)

What 3 investigations and their rationale which are essential before performing a renal
biopsy in this patient?
FBC/ Coag screen & bleeding time - biopsy may be contraindicated if risk of bleeding or in
bleeding disorders
Kidney USS - renal masses, single kidney, size of kidney may affect risk: benefit
ratio and physician's decision to perform biopsy.
U&Es + eGFR - determine degree of kidney impairment
Haematinics - to identify and then follow up with therapy any precipitating causes to anaemia

Which 2 complications should you discuss with the patient when obtaining patient
consent for renal biopsy?
bleeding - perirenal haematoma
clot colic - renal colic due to clot within ureters
arteriovenous fistulas - but typically self resolving

What is the most likely histological diagnosis on renal biopsy in this patient?
lupus nephritis

Which autoantibodies are most commonly detected in SLE? Name 2.


antinuclear antibodies
dsDNA
Smith Antigen

72

Question 62: Orthopaedics/ MSK


Mr J.R., a 65-year-old retired headmaster, is well known to you, having suffered from
increasingly severe osteoarthritis of his right hip for the past 3 years. He had been
very athletic in his youth and played rugby at one time for Glasgow University. He has
come to you as his GP for advice. Mr J.R. says, This hip is really getting me down
now. I have been wondering about a replacement operation. What do you think? You
listen to the patients symptoms and examine him carefully.

Describe two symptoms of hip osteoarthritis that Mr JR may be experiencing


stiffness of joints especially in the morning or after periods of rest
pain - dull/deep thats aggravated by movement
functional ability
range of movements

Describe three signs of hip osteoarthritis that might be elicited on examining this
patient
crepitus
Trendelenburgs sign
active and passive movements - internal + external rotation, flexion
You know that you have really exhausted the three basic principles of the early
management of osteoarthritis with Mr JR.

What are the three basic principles of management of osteoarthritis and by what
means can each be achieved?
Pt education, physical/ occupational therapy, weight , exercise, assistive
devices
simple analgesia - Paracetamol/ NSAIDs
intra articular steroids
You refer the patient to the orthopaedic surgeons and he is seen at the Outpatient
clinic. His hips are X-rayed again.

Describe two characteristic changes that might be found in the appearance of the
patients right hip on such an X-ray.
Loss of joint space
Osteophytes
Subarticular sclerosis
Subchondral cysts

73

Question 63: Haematology


Mrs B.K. is a 76 year old woman who presents to outpatients with a 6-month history of
feeling tired all the time. In view of marked pallor you check her full blood count.

This shows:
Hb 5.4g/dl (Normal range 13.5-18g/dl)
MCV 112 fl (Normal range 80-95 fl)

How would you describe this finding on the blood count?


macrocytic anaemia

You suspect B12 deficiency. What test would confirm this?


Vitamin B12 serum level

What changes do you expect in the bone marrow in B12 deficiency?


erythroid hyperplasia
megaloblastic changes
absence of maturation

What other conditions could produce a high MCV? Suggest 2.


Folate deficiency
hypothyroidism
reticulocytosis
liver disease (+/- due to alcoholism)
myeloproliferative/ myelodysplastic disease

Give 3 causes of B12 deficiency


dietary intake (eg strict vegans)
diminished gastric breakdown of Vitamin B12 from food (eg PPI, acid blockers)
malabsorption from gastrointestinal tract (eg pernicious hb, IBD, bacterial overgrowth
syndromes)

Where is intrinsic factor secreted?


parietal cells of stomach

Where, in the gut, is B12 absorbed?


ileum, small intestine

74

Question 64: Cardiovascular


Palpitations: You are called to the ward to see a 79 year old lady with palpitations. She
has a pulse rate of 200 beats/minute. She is apyrexial. Blood pressure is well
maintained at 130/60. Her ECG is shown.

List 2 causes of palpitations


Vascular (eg aortic aneurysm, A/V fistula, Hb, postural BP)
Inflammation/ infection (c, pericarditis, RhF)
Trauma ( catecholamines)
Anxiety
Metabolic (electrolyte derangement)
Idiopathic/ intoxication
Neoplasm (rare)
Congenital
Drugs (digitalis, aminophylline, nitrates)
Endocrine ( TH, pheochromocytoma, PMS, glu)

What is the dysrhythmia in this case? How would you treat it acutely?
Supraventricular tachycardia
Use vagal manoeuvres
Adenosine 6 mg rapid IV bolus;
if unsuccessful give 12 mg;
if unsuccessful give further 12 mg.
Monitor ECG continuously

She responds and is started on regular digoxin. What advice would you give the
patient about the symptoms of possible Digoxin toxicity. Name 2 factors.
Nausea/ vomiting
abdominal pain
dizziness
headache
confusion
delirium
visual disturbance (blurred/ yellow visions)

How is Digoxin toxicity assessed?


plasma digoxin concentration

75

What ECG changes are commonly associated with Digoxin toxicity?


arrhythmias
- VT/ VF
- SAN block
- AVB
- PVCs
- sinus bradycardia
- depressed conduction
- bigeminal/ trigeminal rhythms/ ventricular bigeminy

What electrolyte imbalance exacerbates Digoxin toxicity?


Hyperkalaemia

76

Question 65: Gastrointestinal/ Hepatic


A 40 year old man attends surgical outpatients for postoperative follow-up. He has
recently had a small bowel carcinoid tumour excised. He complains of feeling
increasingly unwell with diarrhoea, abdominal cramps and occasional episodes of
vomiting. You suspect he has developed carcinoid syndrome.

List 3 other symptoms you would specifically enquire about


Flushing sudden onset with short duration and possibly precipitated by alcohol
Palpitations associated with tachycardia
Oedema has he noticed any swelling of his extremities (RHF associated with
carcinoid)
Wheeze

What biochemical investigation is performed to test, screen for and monitor treatment
of carcinoid syndrome?
Urinary 5-hydroxyindoleacetic Acid i.e. 5-HIAA
Serum Chromogranin A/B (more sensitive but less widely available so not as good an
answer)

What needs to be collected to perform this test?


24hr Urine sample collected in bottle containing acid to maintain pH <3
Blood

Explain why carcinoid syndrome usually occurs later in patients with small bowel
carcinoid tumour than in those with a pulmonary carcinoid.
Carcinoid syndrome is due to excess serotonin entering systemic circulation
In small bowel carcinoid serotonin is released into the hepatic portal system and
so is broken down by the liver before entering circulation; hence symptoms dont
appear unless liver metastases are present or there is already pre-existing liver
damage causing liver function
In pulmonary carcinoid serotonin is released directly into systemic circulation and, though the
lung itself has some capacity to metabolise serotonin, it not enough to prevent earlier
accumulation of serotonin to symptomatic levels

It is difficult to predict on pathological grounds whether a carcinoid tumour will


behave in a malignant or benign fashion. List 2 features of a carcinoid tumour at
presentation that would make it more likely to follow a malignant course.
Site of primary tumour
Depth of local invasion
Metastatic Disease
Size of the tumour

In general, small bowel neoplasms are much rarer than those of the large bowel or
stomach. List 2 other malignant tumours that are relatively common in the small
bowel.
Lymphoma
Gastrointestinal Stromal Tumour (Top 2 = best answers)
77

Adenocarcinoma

Question 66: Cardiovascular


Mr JJ, a 67 year old male returns from holiday to see his GP. While he was away he
developed exertional chest pain which a local doctor diagnosed as angina of effort. He
gives a classical history of ischaemic cardiac chest pain on exercise and, on
examination, you find him to have a blood pressure of 110/85 and a slow rising pulse.
You suspect he might have aortic stenosis and refer him on to the Cardiologists
directly.

Apart from exertional chest pain, name 2 other symptoms from which patient with
aortic stenosis might suffer.
Dyspnoea
Exertional Syncope
Orthopnoea

Please describe 3 features of the murmur you would expect to elicit in a case of AS.
Crescendo-decrescendo pattern peaking in mid-systole
Radiates to the Carotids
Loudest at the 2nd right interspace

Please describe, in less than 50 words, why patients with AS may experience chest
pain.
In AS the valve is narrowed creating a pressure burden on the left ventricle
The LV undergoes concentric hypertrophy in response to the high systolic pressure
The hypertrophied muscle requires more oxygen but does not receive it (Supply-demand
mismatch)
Insufficient oxygen causes ischaemia, which produces chest pain
Concomitant coronary artery disease

What is the most important investigation to identify the cause of Mr JJs problem?
Echocardiogram assess pressure gradient and surface area of valve

List 2 abnormalities you might identify,


Elevated aortic pressure gradient (>50mmHg across the valve)
Reduced valve area (<1cm3)
Reduced left ventricular ejection fraction
LVH

The Cardiologists wish to undertake a cardiac catheterisation, partly for Coronary A.


Disease but also for assessing the degree of stenosis. Mr JJ is due to see the
Dentists in two weeks time. What advice would you give him in the meantime?
Prophylactic Antibiotics
The answer here is most likely prophylactic antibiotics though this is no longer done. Also, it
is important to note that if he had any previous tooth problems, they would have been dealt
with prior to the valve replacement so asking him not to undergo any invasive procedures is
not valid as he wouldnt be anyway.

Question 67: Cardiovascular


You are working on the Coronary Care Unit as a JHO. The Casualty Officer tells you
that a patient, Mr PC, a 35 year old man, has developed signs and symptoms of
78

pericarditis. His arrest page goes off and he is called away and he asks you to sort the
patient out.

List 4 features which would help you confirm the pain is pericardial in origin.
Site: Pain located retrosternally or over left precordium
Onset: Pain is not related to exertion
Character: Pain can be pleuritic, sharp, stabbing or aching (i.e. it is not constricting like
MI/Angina)
Radiation: Pain radiates to the trapezius ridge
E+R Factors: Pain relieved on sitting up or bending forward and worse when lying flat

List 2 signs you would expect to find in Mr PCs case.


Pericardial Rub squeaky/best heard at L sternal edge with patient leaning
forward at end-expiration
c
This scenario is pericarditis and not tamponade meaning you cannot write signs
such as Kussmaulls Sign ( JVP on inspiration indicating limited right ventricle
filling due to RHF), Hypotension or Pulsus Paridoxicus.

The ward sister suggests you do an ECG. List 1 abnormality you would expect to see
if the patient has pericarditis.
Concave upwards ST-segment elevation in all leads bar aVR/V1
PR Depression
Clearly Mr PC is in pain.

What would be the first-line treatment of choice?


NSAIDs e.g. Ibuprofen
We initially had the above answer for this question, but the Cardiology consultant informed
us that this answer was IV Morphine Sulphate because you wanted immediate (i.e. IV) and
potent (i.e. morphine) pain relief. However, following an faq on vale/further discussion weve
changed it back to Ibuprofen.

Give 2 side-effects that you would warn him about because they may cause him
symptoms
Gastro-intestinal Disturbances e.g. discomfort, nausea, diarrhoea and rarely
bleeding/ulceration
Hypersensitivity Reactions i.e. rashes, bronchospasm

The FY2 returns from his arrest and asks you what other causes of pericarditis there
are apart from viral disease.
Neoplasm
Idiopathic
Myocardial Infarction
Autoimmune Disorders e.g. RA
79

Metabolic Disorders e.g. Uraemia


Iatrogenic e.g. Radiotherapy, Cardiac Surgery, PCI
Uraemic - Other Infective e.g. Bacteria

80

Question 68: Cardiovascular


DY is a 78 year old male with a history of a large anterior myocardial infarction three
years ago. He presents with a three week history of fatigue, ankle swelling, decreased
exercise capacity and shortness of breath. On examination there was pitting oedema,
cardiomegaly, raised jugular venous pressure and crackles at the lung bases. ECG
showed sinus rhythm and echocardiography revealed severe left ventricular systolic
dysfunction with ejection fraction of 26%.

What is the pathophysiological significance of the raised jugular venous pressure and
crackles at the lung bases?
The raised JVP indicates systemic venous congestion, which suggests Right Heart Failure is
present
The crackles indicate pulmonary congestion, which suggests Left Heart Failure is also
present
Hence, the patient has biventricular cardiac failure with pulmonary oedema

Which class of diuretic would you choose and why?


Loop - trigger prompt onset of significant diuresis

Spironolactone can be used as an effective way of preventing diuretic-induced


hypokalaemia. Name 1 other drug that has a similar affect.
Eplerenone (aldosterone antagonist like spironolactone)
Amiloride (K-sparing diuretic)
Triamterene (K-sparing diuretic)

What additional benefits may accrue to the patient as a result of using


spironolactone?
Combats effects of Hyperaldosteronism
Reduced Mortality

Digoxin, a beta-blocker and dobutamine were all considered for initial treatment but
rejected. Give 1 reason for rejecting each of these.
Digoxin the patient has sinus rhythm (no evidence of benefit)
-Blocker contraindicated in acute heart failure
Dobutamine inotropic agents should only be used to treat acute
decompensation

DY was finally started on an angiotensin converting enzyme inhibitor (ACE inhibitor).


Name 2 precautionary measures that you would warn Mr DH about and that should be
taken on starting this medication.
Warn about side effects e.g. first-dose hypotension, cough, hyperkalaemia, renal impairment
Start at a low dose and titrate up
Check Urea and Electrolytes before initiating treatment (? hyperkalaemia or renal
dysfunction)
Check patient is not taking drugs that have interactions with ACE-I e.g.
Ciclosporin ( risk of K+)

81

Question 69: Dermatology


A 35year old woman consults her General Practitioner because she has developed a
rash that she thinks is similar to her brothers psoriasis.

What features in the skin will support a diagnosis of psoriasis. Suggest 4.


Characteristics well-demarcated, silver scale, salmon pink plaques
Distribution extensor surfaces, scalp, torso
Nail Changes onchylosis, pitting

Under what conditions may psoriasis make patients systemically unwell?


Erythrodermic Psoriasis
Generalised Pustular Psoriasis
side effects of systemic drugs used in treatment of psoriasis

The patient has been using her brothers topical corticosteroid cream. Why are topical
corticosteroids not used a first-line therapy?
Side effects i.e.:
Local skin thinning, striae, easy bruising/fragility, risk of skin infection,
steroid acne
Systemic adrenal suppression, Cushings Syndrome
Risk of developing unstable psoriasis with potent corticosteroids when treatment is stopped
Note that in practice, topical steroid combinations are often used as a first line e.g. with
calcipitriol or tar

Name 2 topically applied treatments that should be used as first-line treatment.


Emollients e.g. soap substitutes, bath additives, creams/ointments
Vitamin D Analogues e.g. calcipotriol (Dovonex), calcitriol, tacalcitriol
Coal Tar e.g. psoriderm
Less Commonly dithranol 0.1% and tazarotene Gel

Apart from UV phototherapy, name 1 second-line oral drug that a Dermatologist might
offer and state an important side effect associated with its use.
Drug Methotrexate, Ciclosporin, Acitretin, Fumaric Acid
Side Effect Pancytopenia/ Liver Cirrhosis

82

Question 70: Dermatology


David, a 16 year old boy, is referred to dermatology by his GP. He has extensive red,
raised oval plaques with white scaling on his scalp, knees and elbows. The plaques
are itchy and he is very self-conscious. You make the diagnosis of moderate classic
plaque psoriasis.

Name 3 other types of psoriasis


Guttate
Flexural
Pustular
Erythrodermic
OR:
Scalp
Nail
Hand
Foot
though probably the actual types

List 2 clinical signs you would seek on examination of the skin to support your
diagnosis
Characteristics well-demarcated, silver scale, salmon pink plaques
Distribution extensor surfaces, scalp, torso
Nail Changes onchylosis, pitting

A number of environmental factors can trigger an outbreak of psoriasis. Name 2 drugs


that may make psoriasis more active.
Lithium
Anti-Malarials e.g. Hydroxychloroquine
-Blockers
NSAIDs

List 4 nail changes that may occur in psoriasis


Onycholysis
Pitting
Subungual Hyperkeratosis
Beaus Lines (transverse ridging)
Nail Thickening (onychauxis)
Yellow Discolouration
Splitting and Brown Oil Spot Sign (nail plate crumbling)

What types of topical treatments could you start David on? Name 4.
Topical Steroids of mild-mod potency e.g. Hydrocortisone Ointment 1%
Emollients
Vitamin D Analogues
Tar-based Creams e.g. Psoriderm
Less Commonly Dithranol and Tazarotene

You see David frequently over the next year. His psoriasis did not respond well and
you now class it as severe. He is very anxious and you decide to start him on
83

systemic treatment. Your consultant prescribes Acitretin, an oral retinoid. Who would
you not give this drug to and why?
Who women of child-bearing age Also: patients with hyper-lipidaemia / liver
problems
Why teratogenic Exacerbates lipids/ Risks further impairment of liver
function

84

Question 71: Infection


You are a junior house officer on a medical ward and are asked to see a 34 year old
lady who is receiving parenteral nutrition. She has a temperature of 38C and a
tachycardia.

Name three conditions where patients commonly receive TPN.


Short Bowel Syndrome
Active Inflammatory Bowel Disease
Bowel Obstruction

What complication of TPN do you suspect in this case?


Sepsis

Outline 2 initial steps in the management of this patient.


Take a line AND peripheral culture
If central venous line-related sepsis is suspected remove the line
Prescribe antibiotics after organisms have been identified

Resistant organisms are an increasing problem in hospital-acquired infections. Name


4 steps that are taken to minimise the risks of developing infections with organisms
such as MRSA?
Screening and treating MRSA carriers
Hand wash before and after seeing patients
Use disposable gloves while handling open wounds
Keep the hospital environment clean
Isolate patients with known/ suspected MRSA infection

The patient subsequently develops a painful swollen right arm. What complication do
you suspect and how should it be managed?
Complication Axillary Vein Thrombosis
Management Give Heparin

85

Question 72: Gastrointestinal/ Hepatic


A 40 year old man presents with a 10 year history of heartburn as a main symptom.

List the 2 likely differential diagnoses of patients with heartburn.


Gastroesophageal Reflux Disease
Duodenal or Peptic Ulcer

As his GP, list 2 lifestyle measures that you would suggest he takes into account as
part of the treatment.
Eat small, frequent portions of meal at regular times
Reduce alcohol intake
If appropriate attempt to lose some weight
If appropriate attempt to cut down smoking

What 2 therapeutic measures would you suggest?


Proton Pump Inhibitior: e.g. Omeprazole
H2 receptor antagonist: e.g. Ranitidine, Cimetidine

A year later he represents after following all your initial management and his
symptoms are as troublesome as before. You refer him to a gastroenterologist. List
three investigations likely to be necessary.
Urea breath test
Oesophogastroduodenoscopy + biopsy
Oesophageal pH or Manometry
Biopsies from his distal oesophagus demonstrate extensive intestinal metaplasia

What is the diagnosis?


Barretts Oesophagus columnar metaplasia of squamous cells in distal
oesophagus

Why is it significant?
Barretts oesophagus can progress to oesophageal adenocarcinoma

86

Question 73: Infection


You are the on-call FY1 and have been asked to see a 78 year old man, Mr EE, in
Casualty who has been sent up by his General Practitioner from his residential home.
He is known to have ischaemic disease and had a myocardial infarction 4 years ago
and has now had diarrhoea for more than one week and is unwell. A stool sample sent
by the nurses in the residential home has grown Salmonella.
Examination shows him to be feverish (37.9C), slightly confused and with a blood
pressure of 105/70. He has minimal abdominal signs, apart from some tenderness in
the left iliac fossa.
His initial investigations are as follows:
Sodium 130 mmol/L (NR 135/145mmol/L)
Potassium 3.1 mmol/L (normal range 3.5 5 mmol/L)
Urea 28.2 mmol/L (normal range 2.5 6.7 mmol/L)
Creatinine 182 mmol/L (normal range 70 150 mmol/L)
Random Blood Glucose 7.2 mmol/L (fasting glucose 3 5.5 mmol/L)

List 2 biochemical abnormalities that you have identified in these results and provide
an explanation as to why each might be relevant, given the above history.
Raised Urea/Creatinine indicates dehydration and need for fluid resuscitation
Hypokalaemia indicates significant diarrhoea and may cause
arrhythmias/altered nerve conduction
There are no beds available within your Infectious Disease Unit. You decide to admit
Mr EE.

What advice would you give to the nurses regarding his infectious state?
Salmonella can spread directly from person to person so barrier nursing is needed
Following Salmonella infection a patient continues to carry and secrete organisms for several
weeks

What type of intravenous fluid would you prescribe given his biochemical results?
Hartmans or equivalent crystalloid solution with Potassium

Over what period of time would prescribe the first 500ml bag?
Rehydrate at least half the calculated deficit (based on degree of reduced intake and
diarrhoeal fluid loss) over 3-4 hours, so first 500ml usually given over 30minutes to 1 hour

What antibiotic therapy would you prescribe for his salmonella?


For patients with risk factors of bacteraemia (e.g. <3 months, >50 years, immunosuppressed)
Fluoroquinolones (e.g. Ciprofloxacin or Ceftriaxone) are given

Despite your best efforts, the patient dies in the early hours of the morning and you
are called to confirm his death. Give 2 observations that you would use to confirm
death.
Palpation of all major pulses, all absent
87

Fixed pupil dilatation


Auscultate heart for >5 minutes, no heart sounds
No respiratory effort for 1 minute
The following morning, after discussion with your FY2 doctor colleague, you are
anxious about the circumstances of the death and decide to call the Procurator Fiscal
before signing the death certificate. Please list 4 circumstances where it is mandatory
to inform the Procurator Fiscal about an adult death.
Sudden death
Deaths related to neglect or complaint
Drug-related death
Food poisoning
Death during administration of GA
Legionnaires

88

Question 74: Rheumatology


A 50 year old woman has had Raynauds phenomenon and a photo sensitive skin rash
for about 12 years and you have made a clinical diagnosis of systemic lupus
erythematous (SLE). She has recently developed ankle oedema and has proteinuria on
urinalysis. You suspect she may have developed nephrotic syndrome.

What 2 laboratory results would confirm the clinical diagnosis of nephrotic syndrome?
Give figures.
Proteinuria (>3g/24hrs)
Hypoproteinaemia (serum albumin <25g/L)

List 3 investigations that are essential before performing a renal biopsy in this patient.
Renal US (ensure patient has 2 normal sized non-obstructed kidneys)
FBC
Clotting Screen
U&Es
Group and Save
Early morning protein:creatinine ratio

Name 2 complications you should discuss with the patient when obtaining patient
consent for renal biopsy.
Bleeding (peri-renal haematoma occurs in 65-80%; others = Micro/macroscopic Haematuria
Pain typically in flank and possibly radiating to shoulder
Introduction of infection
Mortality (0.1%)

What is the most likely histological diagnosis on renal biopsy in this patient?
Membranous Nephritis + In-situ Immune Complex Deposition

Name 2 autoantibodies that are most commonly detected in SLE.


ANA
Anti-dsDNA

89

Question 75: Gastrointestinal/ Hepatic


You are a junior house officer on a surgical ward during a night shift. The nurses call
you to see a patient who has had a major abdominal operation that day, whose blood
pressure has fallen to 90/70. The Oxygen saturation is 98% but they are concerned
that the patient has not passed urine following surgery.

List 4 clinical signs you would use to assess the patient and explain why these would
be valuable.
JVP surrogate for right atrial pressure so fluid overload and fluid
depleted
Skin Turgor measure fluid state
Mucous Membranes measure fluid state
HR tachycardia may be precipitated by hypovolaemia
Temperature <36/>38 may indicate sepsis
Palpable Bladder may be an obstructive cause for AKI
Respiratory Exam may reveal pulmonary oedema
Your FY2 suggests that it may be useful to put in a central venous pressure line.

How would this be used to assess the patients status?


CVP can be used to monitor volume status in a patient
It is used to estimate right ventricular filling pressure (i.e. preload)
In this patient it would be useful in determining the efficacy of a fluid challenge

What is an estimate of the volume of urine needed to be passed per hour to ensure
that oliguric renal failure is less likely to occur?
0.5ml/kg/hr

How can this be assessed?


Catheterise the patient and measure hourly urine output using a fluid balance chart

If the urine output fails to reach this figure, what course of action should be taken?
Give fluid challenge and measure the response
Seek senior review +/- HDU/ICU Support depending on patient status

90

Question 76: Neurology


Mr FH is a 68 year old man admitted under your care as an emergency. One year
previously he had a lung resection for cancer. He has done well since then, but for the
past month he has had back pain, and for the last 3 days has been unable to walk
because of weakness and numbness in both legs. You consider that he may have a
secondary deposit in the spine.

What other functional disturbance is likely to be present on inquiry?


Sphincter disturbance bladder and bowel incontinence
Saddle Anaesthesia
Sensory Disturbance

Your clinical examination suggests the presence of an upper motor neurone lesion.
List 4 features that would suggest this is the case.
Increased tone (clasp knife)
Clonus
Hyperreflexia
Extensor plantars
An MRI scan confirms your diagnosis with a secondary deposit in the body of the 11th
dorsal vertebrae. The radiologist contacts you to point out that there is pressure from
this lesion on the thoracic spinal cord.

Suggest 3 urgent treatment that should be considered to help the symptoms of leg
weakness.
IV Dexamethasone
Local Radiotherpy
Surgical Decompression Laminectomy

Further investigation shows a raised serum calcium level. Give 2 possible


explanations for this.
Bony metastases lysis of bone and activation of osteoclasts increased
calcium release
PTHrP (secreted in some lung cancers) calcium uptake from gut and
increased bone turnover

91

Question 77: Neurology


Mr B.B. is a 73-year old man admitted for routine hernia repair. You are the FY1 and
while clerking him his wife tells you that his voice has changed, his words are
becoming harder to understand and that he mumbles. His wife says his symptoms
seem like Parkinsons disease.

List 2 other features you would ask about in the history to confirm the diagnosis.
Slow Movement
Handwriting changes (Micrographia)
Tremor
Difficulty turning around

What other features will you look for on examination? List 6.


Pill Rolling Tremor
Bradykinesia
Cog-wheeling or Lead-pipe Rigidity
Low Blink Rate
Drooling
Postural Instability
Masked Face
Shuffling Gait
Stooped Posture
Conjugate Gaze Disorders (either jerky pursuit or falling short of intended target)
Confirm Altered Speech Hypophonia (volume) and Hypokinetic Dysarthria
(mumbling)

What pathophysiological change underlies this disease? Give a brief reply.


Essentially, the pars compacta of the substantia nigra undergoes progressive neuronal
degeneration
This degeneration involves loss of the dopaminergic neurons of the nigrostriatal pathway
coupled with intracytoplasmic Lewy Body formation
The overall result is a loss of dopaminergic input to the basal ganglia effectively placing it in
an inhibitory state
The usual treatment for this condition is a combination of 2 drugs given as a single
preparation.

What are the 2 drugs?


L-dopa
Carbidopa
Combined e.g. Sinemet

Why is the combination necessary?


To prevent peripheral metabolism of L-dopa and in doing so minimise the side-effects while
maximising its efficacy.

Name 2 classes of drugs (with one example of each) that can cause these symptoms.
Typical Antipsychotics e.g. Haloperidol
92

Anti-emetics e.g. Metoclopramide

93

Question 78: Orthopaedics/ MSK


Mrs MS, a 53 year old lady presents to A&E with back pain which has been there for
the past week. She tells you that she has flu-like symptoms but is normally fit and
well. On further questioning she tells you she is an ex-smoker, has eczema and has
recently lost stone in weight in the last month. On examination she is tender,
particularly over the upper lumbar area but neurological system examination of the
lower limbs is normal. You note she had an early menopause at the age of 40.

Apart from osteoporosis what would your top 2 differential diagnoses be in this lady
and why?
Osteomyelitis
Mechanical Back Pain
Cancer (weight loss and smoking)

List 4 risk factors that you would enquire about in a patient in whom the diagnosis is
suspected to be osteoporosis?
Hx of steroid therapy
Age of menarche/menopause whether or not she used HRT (HRT decreases risk)
Family Hx e.g. of hip fracture
Bone protection i.e. Calcium Intake, sun exposure
Social Hx smoking and alcohol use
PMH prolonged period(s) of immobility
Occupational Hx Space Travel

An X-ray confirms a crush fracture at L1. What key investigation would you now wish
to perform to confirm the diagnosis of osteoporosis?
DEXA scan

Mrs MS has been receiving treatment with hormone replacement therapy for the past
five years for post-menopausal symptoms. Bearing in mind recent research
developments, what advice would you now give her about this form of treatment?
After 5Y: risk breast cancer, stroke, DVT/PE and CV disease BUT risk is still low.
risk colon cancer and hip fracture

What advice and treatment would you now give to the patient on discharge? List 4
separate items.
Alendronic acid 70mg once weekly
Cholecalciferol 600 units OD
Regular exercise strength and balance training
Avoid excess alcohol and cut down smoking
Increase dietary calcium intake and sun exposure/vitamin D intake
Ensure there are no trip hazards around the house

94

Question 79: Infection


33 year old female asylum seeker from Ethiopia presents with a 3-month history of
cough, weight loss and night sweats. On examination there are coarse crackles at the
apex of the right lung.

What is the most appropriate imaging investigation that should be carried out at this
stage?
CXR

List 3 further non-invasive investigations that would be appropriate and how each
would help you in making the diagnosis?
Sputum Culture and Sensitivity (AAFB) identify causative agent
AAFB Sputum Smear (Ziehl-Nielsen/Auramine Stain) rapidly identify AAFB if
present
Sputum Nucleic Acid Amplification Test for M. Tuberculosis rapidly identify M. TB
if present

What is the most likely diagnosis in this individual?


Pulmonary Tuberculosis

List 2 other differential diagnosis that would be important to exclude.


Bronchial Carcinoma
Sarcoidosis
Fungal infection

What is the most appropriate immediate course of action to be suggested to the nurse
manager when this patient is admitted?
Isolate the patient and initiate barrier nursing

Once your most likely diagnosis is confirmed, who should be informed immediately
and why?
Who Public Health Consultant
Why to initiate contact tracing for prophylactic vaccination and, secondly, so
they know how the disease is behaving in the public for epidemiological studies

95

Question 80: Rheumatology


A 34 year old woman has a CXR for insurance purposes. She is asymptomatic. The Xray show bilateral hilar lymphadenopathy and sarcoidosis is suspected.

List 3 clinical features of sarcoidosis that you would specifically look for on your
initial clinical examination?
Arthralgia + OTHERS:
Erythema Nodosum
Lymphadenopathy
Others:
Facial Palsy
Photophobia
Red Painful Eye
Wheezing
Rhonchi
Lupus Pernio

List 2 investigations (other than biopsy) that might help confirm the diagnosis.
FBC leukopaenia / anaemia
Serum Calcium hypercalcaemia
Purified Protein Derivative of Tuberculin exclude TB
ECG identify cardiac involvement
NOT CXR SINCE IT IS MENTIONED ABOVE

The above investigations are inconclusive and a biopsy is performed.


What is the characteristic microscopic finding in sarcoidosis?
Non-caseating Granulomas

List 2 other differential diagnosis that would be important to exclude.


Bronchial Carcinoma
TB

What is the most appropriate immediate course of action to be suggested to the nurse
manager when this patient is admitted?
Isolate the patient
NOTE: We have been assured question will appear in exams. The idea behind the question
is that you should isolate the patient as a precaution in case it transpires that the bilateral
hilar lymphadenopathy is due to TB. However, we were told that in practise this would just
never happen as the odds of that being the case in the UK are so slim. Hence, the question
should not make it through the 2012 selection process.
At your initial assessment she has no signs of involvement of other organs and you
decide that no specific treatment is required. You see her again in 3 months but on
that occasion she tells you she has a dry non-productive cough and becomes
breathless on fairly mild exertion.

96

List 3 cardiovascular or respiratory complications of sarcoidosis that might explain


these symptoms.
Cardiomyopathy
Pulmonary Hypertension
Congestive Heart Failure
Pulmonary Fibrosis

97

Question 81: Orthopaedics/ MSK


A 72 year old obese lady presents with a 6 month history of increasing right knee pain.
She has no other musculoskeletal complaints. There is no relevant past medical
history

What is the most likely diagnosis?


Osteoarthritis

List 2 features of her history obtained on further questioning, that would be important
to take into consideration when assessing the severity of her symptoms?
Function is it impacting her daily life/ does it restrict her getting about
Pain how severe is the pain

List 4 features you would look for on examination, which would support your
hypothesis?
3 Js, 3 Ms + Bony Swelling:
Joint tenderness
Joint instability
Joint effusion
Movement limited
Movement crepitus
Muscle wasting
Bony swelling

List 3 broad types of non-surgical management.


Weight Loss keep BMI below 25
Exercise moderate exercise to muscle strength and joint load
Physiotherapy maximise functionality
Occupational Therapy behavioural and home adaptations to cope with
condition
Orthotics correct footwear to support and inappropriate loading

98

Question 82: Rheumatology


A 42 year old female presents with a 6 month history of painful hands and wrists and a
one week history of sore red watery eyes which are tender to touch. You think she
may be suffering from rheumatoid arthritis and send her for an X-ray of her hand and
take a blood sample.

RA has a familial pattern. What is the HLA type associated with its development?
HLA-DR4/DR1

What X ray findings would you suspect if this was a case of rheumatoid arthritis?
JLS-P: (like the band but with a P)
Joint Erosion
Loss of joint space
Soft tissue swelling
Peri-osteal osteoporosis
Routine blood investigations show an increased CRP and a reduced haemoglobin.

Explain these findings.


RA is an inflammatory condition characterised by synovitis
Inflammed synovium releases cytokines like IL-6
IL-6 triggers production of acute phase proteins in the liver (e.g. CRP) to clear
dead/damaged cells
Inflammatory cytokines also iron metabolism and sensitivity of bone marrow to
EPO Hb
Bone marrow may also be suppressed in RA due to anti-rheumatoid drugs e.g. Methotrexate
NSAID use may cause GI Blood loss Hb
You suspect her eyes problems are related to her rheumatoid disease

List 2 ocular complications associated with RA.


Scleritis
Episcleritis
Keratoconjuctivitis
Your initial clinical impression that the patient is suffering from rheumatoid arthritis) is
confirmed by a rheumatologist, who, with the general practitioner, will co-manage the
medical treatment that has been commenced.

List 3 other health professionals that will be involved in the long-term care of this
patient and outline what each would contribute.
Orthopaedic Surgeon total joint replacement, synovectomy, excisional
arthroplasty
Physiotherapist dynamic exercise therapy, hydrotherapy, TENS
Occupational Therapist advice and home adaptations to help manage daily
living i.e. washing etc
Orthotics splints
Dietetics minimise risk of anaemia

99

Question 83: Orthopaedics/ MSK


A 65-year old obese lady presents with increasing pain in the right groin. The pain is
worse on walking and at the end of the day and she volunteers the information that
she also gets pain in the right knee at a similar time.

What is the most likely diagnosis and which is the joint involved?
Osteoarthritis of the Right Hip

You decide to arrange an X-ray. Name 4 features that you would be likely to see in this
case.
LOBS:
Loss of Joint Space
Osteophyte Formation
Bone Cysts
Sub-articular Sclerosis

What is the most appropriate form of management? Name 2 aspects and mark the one
you consider to be most important.
Non-pharmacological (Most Important) weight loss, exercise, physio, OT,
orthotics
Pharmacological analgesia (e.g. paracetamol/NSAIDs) +/- intra-articular
corticosteroids
Surgical arthroplasty is typically best though osteotomy and arthrodesis may also be
used

Six months later she continues to complain of pain, which is not relieved by your
primary treatment. The pain now wakes her at night and you notice a 5 fixed flexion
deformity. What procedure would now be indicated?
Total Hip Arthroplasty
Her daughter asks you about long-term complications of this surgical procedure.

Describe 2 long-term complications that you would tell her about. (Less than 50
words)
Chronic pain and stiffness can be a problem
However, if her mother is compliant with rehabilitation programme this should be minimised
There may be a need for repeated surgery as the joint gradually loosens with activity
Late infection may occur and require antibiotics/revision surgery
As with all operations, death is a potential long-term complication
Early Complications DVT, PE, septic arthritis, limb length discrepancy, medial
acetabular wall rupture

100

Question 84: Orthopaedics/ MSK


A 56-year old female presents with pain in her right hand. The pain is worse at night
and radiates through her forearm. You consider that Carpal Tunnel Syndrome (CTS) is
a possible diagnosis.

In addition to pain, what other symptoms, in connection with her hand, would you
enquire about? Name 2 symptoms.
Paraesthesia median nerve territory i.e. thumb, index, middle and radial half of
ring finger
Decreased Sensation
Weakness of thumb abduction
Clumsiness of hand

List 4 features on physical examination or testing that would help confirm your
diagnosis.
Reduced sensation in median nerve territory
Wasting of muscles of thenar eminence (OAF)
Weakness of abduction of thumb (abductor pollicis brevis)
Positive Phalens test (reproduction of symptoms on full flexion of wrists > 1min)
Positive Tinels test (paraesthesia on tapping over course of median nerve)

Name 6 conditions associated with CTS


Inflammatory RA
Renal Chronic Renal Failure, Dialysis (AV fistula causes pressure in carpal
tunnel nerve ischaemia)
Endocrine Pregnancy, Hypothyroidism, Acromegaly, DM

What investigation could confirm the diagnosis of CTS?


Electromyography nerve conduction in the median nerve

List 2 possible treatment of CTS


Conservative Wrist Splint, Intra-articular Joint Injection, Avoiding precipitating
activities
Surgical decompression surgery i.e. division of flexor retinaculum

Name 2 symptoms and signs that differentiate ulnar nerve entrapment from CTS.
Distribution numbness and paraesthesia on ulnar half of ring finger and little
finger
Wasting hypothenar eminence and interossei
Weakness of abduction of fingers i,e, cant cross fingers due to weakness of
interossei
Froments sign flexion of PIP joint while grasping piece of paper between thumb
and index finger due to weakness of adductor pollicis
Poor flexion of 4th and 5th DIP joints with weakness of wrist flexors

101

Question 85: Endocrinology


You are asked to see a patient who has lost 5kg in weight and has recently become
diabetic. He has a longstanding history of alcohol abuse and has been diagnosed with
chronic pancreatitis in the past.

Other than pancreatic insufficiency, what is the main symptom of chronic


pancreatitis?
Chronic epigastric pain radiating to the back, which is typically exacerbated by alcohol

What are the 3 main cell types in the islets of Langerhans and what hormones do they
secrete?
Alpha Glucagon
Beta Insulin
Delta Somatostatin

What are the 2 main symptoms of pancreatic exocrine insufficiency?


Weight loss
Steathorrea

Name 3 digestive enzymes secreted by acinar cells.


Target Protein Trypsinogen, Chymotrypsinogen, Proelastase,
Procarboxypeptidase A/B
Target Carbs Pancreatic Amylase
Target Fats Colipase, Pancreatic Lipase
DNA/RNA etc Ribonuclease, Deoxyribonuclease, Phospholipase A2
Cholesterol Bile-Salt-Acid Lipase, Cholesterol Ester Hydrolase

What treatment is available for management of exocrine insufficiency


Creon (pancreatic enzyme supplements)
PPI should also be given to minimise Creon destruction in stomach
Fat soluble vitamins e.g. Multivite
Though the above 2 are relevant neither is correct as Creon is what actually corrects the
insufficiency

102

Question 86: Gastrointestinal/ Hepatic


A 54 year old man presents with a 3 week history of painless jaundice. He has lost 2kg
in weight and has pruritus.

Name two additional features that you would expect to elicit if his jaundice is
obstructive in nature?
Dark urine
Pale Stools (steatorrhea)
You had recently prescribed a course of antibiotics and are concerned that he may
have a drug-induced jaundice.

Name 3 mechanisms by which antibiotics may cause jaundice and for each cause give
one example of a drug that may be responsible.
Pre-Hepatic: Impaired Bilirubin Uptake (e.g. Rifampicin) unconjugated
bilirubin jaundice
Intra-Hepatic: Induce Liver Failure (e.g. Rifampicin) excretion of bilirubin
bilirubin jaundice
Post-Hepatic: risk of gallstones (e.g. Flucloxacillin) blocks duct excretion
jaundice
Allergic Haemolytic Anaemia (e.g. Cephalosporins) RBC breakdown
bilirubin jaundice

An abdominal ultrasound is arranged. What findings would lead you to conclude that
he has extrahepatic biliary obstruction?
Dilatation of the Common Bile Duct/Common Hepatic Duct

Name the two main causes of extrahepatic biliary obstruction in a man of this age?
Neoplastic Disease of pancreatic head, ampulla or bile duct (i.e.
Cholangiocarcinoma)
Choledocolithiasis (common bile duct stone)

103

Question 87: Gastrointestinal/ Hepatic


A 72 year old lady presents to the Accident and Emergency department with a history
of haematemesis and collapse. She is resuscitated and taken for immediate upper GI
endoscopy where she is found to have a large bleeding duodenal ulcer.

What drugs are commonly responsible for this complication in elderly patients?
NSAIDs e.g. Ibuprofen
Antiplatelets/Anticoagulants e.g. Warfarin

Name two tests available to identify H. Pylori as a cause for peptic ulcer disease?
Urease Breath Test
H. Pylori Stool Antigen Test or Serology
Gastroscopy and CLO test or histological assessment

Apart from blood transfusion list two treatment options for the management of the
acute bleed in this patient.
Resuscitation
Endoscopic Haemostasis e.g. Thermal Coagulation, Mechanical Clips, Adrenaline Injection
PO or IV PPI e.g. Omeprazole.
We have been told this is a badly worded question. It should really just be asking What is
the most appropriate management for this patient? with the answer being Resuscitation
followed by Endoscopic Haemostasis. Also, with regards to IV PPI, it is expensive and there
is limited evidence as to its efficacy. IV PPI (normally Omeprazole) is indicated following
successful endoscopic haemostasis (commonly referred to as the Hong Kong protocol). In
conclusion there is a role for PPI, but this should be given orally for the majority of patients.

Following successful control of the bleeding, she is returned to the ward and
commenced on an infusion of omeprazole. What is the mechanism of action of this
drug?
Proton Pump Inhibitor
Irreversibly blocks hydrogen/potassium ATPase in parietal cells
Result is significantly reduced H+ secreted HCl, which results in an increase
in gastric pH
pH is beneficial because it results in improved platelet function

What hormone is secreted by the gastric mucosa? What cell type is responsible?
Gastrin, G-cells

104

Name three conditions associated with increased plasma levels of this hormone.
Zollinger-Ellison Syndrome
Atrophic Gastritis
Pernicious Anaemia targeting parietal cells ONLY
Peptic Ulcer Disease
Atypical Site of Gastrin Secretion e.g. Meckels Diverticulum with Gastric Mucosa or
Paraneoplastic
There are 2 types of Pernicious Anaemia. One targets the parietal cell H/K ATPase pump (causing acid gastrin) with the other blocking B12/Intrinsic
Factor binding or absorption (and hence never affecting gastrin levels).

105

Question 88: Gastrointestinal/ Hepatic


A 48 year old publican is admitted as an emergency under your care with an episode
of severe upper abdominal pain and vomiting of acute onset. He has no previous
relevant history. On examination you elicit the signs of peritonitis, mainly in the upper
abdomen. Urgent investigations reveal a serum amylase of 2,800 IU. There is no gas
under the diaphragm on plain radiology.

What is the most likely diagnosis?


Acute Pancreatitis

Name 2 of the commonest identifiable causes of this condition


Alcohol
Gallstones
The acute episode settles with pain relief and intravenous fluids. You request an
ultrasound investigation, which demonstrates a common bile duct with a diameter of
12mm containing a rounded mobile 5 mm echogenic focus.

What techniques are currently available that would permit you to image the bile duct
using appropriate contrast?
Endoscopic Retrograde Cholangiopancreatography
Percutaneous Transhepatic Cholangiography
Operative Cholangiography
HIDA
NOT Magnetic Resonance Cholagiopancreatography as it does not use contrast
NOT CT Pancreas because although it uses contrast, it is venous and never enters the bile
duct
We have been told that this question is terrible due to the contrast component. The patient
has had an episode of acute pancreatitis and has imaging, which suggests
choledocolithiasis. With this in mind, the only appropriate way to proceed is with ERCP.
If the US had not diagnostic (for example if it had demonstrated minimal dilatation of the
CBD with no choledocolithiasis) then MRCP is the most appropriate test.
Alternatively the patient could have a laparoscopic cholecystectomy with intra-operative
cholangiography, but this relies on the operator then having a strategy to manage
choledocolithiasis surgically, so most surgeonswould ensure that the duct was clear with
ERCP or MRCP prior to surgery. There is no real role for CT in the imaging of the biliary tree
as the images produced are not as good as on MRCP. The only situation where a patient
would get a CT over an MR would be if the surgeons anticipated encountering neoplastic
disease of the pancreatic head/CBD HIDA is a functional investigation and I would not
consider it useful/relevant in this situation.

106

What definitive surgical management options would you like to discuss with your
patient?
ERCP + Sphincterotomy and a combination of balloon trawl of CBD and/or stent
Follow above with laparoscopic cholecystectomy if appropriate, OR
Laparoscopic cholecystectomy with intra-operative cholangiography and laparoscopic CBD
exploration
This question is asking about specific management of Choledocolithiasis.

107

Question 89: GU
A 25 year old man presents to his GP with an enlarged non-tender testis. A tumour is
suspected.

List 2 investigation that might be helpful in confirming the diagnosis


Imaging USS (the most important and relevant investigation)
Blood Tests -fetoprotein, -HCG, LDH (that order = most to least useful)
Staging Chest/Abdo/Pelvis CT (in any cancer it should be chest/abdo/pelvis)
The results of the tests confirm that the testis contains a tumour and an orchidectomy
is planned.

What is the common type of testicular tumour in this age group?


Teratoma (Peak age 20-30 years; Seminoma peaks at 30-40)
In order to identify the need for further treatment, staging is undertaken. Imaging of
the chest and abdomen is carried out for staging.

What is the most common site of metastases from these tumours?


Lymphatic spread is the most common cause of metastasis and commonly occurs through
spermatic cord lymphatics to the retroperitoneal lymph node chain
Lung (most common solid-organ metastases)

List any 4 other possible causes of scrotal swelling.


Testicular - Torsion, Epididymo-orchitis, Cancer (he has it so no mark for it)
Scrotal - Epididymal cyst, Hydrocoele, Varicocele, Inguinal hernia

Older men (e.g. over 60) are generally affected by different testicular tumours from the
younger age group. Name 2 types of neoplasm in the testis that would be likely to
present in the older age group.
Lymphoma
Interstitial Tumour

His girlfriend comes to see you and asks what the prognosis would be for children
post-surgery if the testis needs to be removed. What principles would you use to
guide what you tell her?
Confidentiality i.e. as bf has capacity you are legally obligated to follow his wishes and since
he has not made it clear that she can be informed about specifics of his rx you are unable to
discuss his case with her without him being present and consent being given
Truthfulness/Honesty: as a general rule in cases where an orchidectomy is required, the
other testicle compensates for the loss. In the unlikely situation both have to be removed,
sperm is 'harvested' and stored for future use
Non-maleficence: you have no idea what the impact of telling her he either will or wont be
able to have children may have and so, it is best to speak in general terms.
This question is absolutely dreadful. The first point is by far the most important. Anything else
within reason will get the marks.
108

Question 90: Cardiovascular


Mr. B.Y. is an 80 year old man who has pain in his feet at night while in bed. He is a
smoker and has diabetes mellitus.

You think he has rest pain secondary to arterial disease. Give 3 features from the
history which would support the diagnosis.
Elderly
Male
Smoker
Known DM
Hypertension
Hypercholesterolaemia
Though neither of the above are specifically mentioned, the surgeon informed us that, as
final year students, we are expected to be able to identify factors such as these i.e. 99% of
80 year olds will have hypertension and hypercholesterolemia and hence, it is acceptable to
write these.

Give 4 features on examination which would support your diagnosis.


Absent or diminished peripheral pulses
Cold/shiny/scaly skin
Pallor of peripheries in particular on elevation
Hair loss from the symptomatic limb
Ulceration/gangrene
Nail changes
NOT Muscle atrophy as uncommon and more likely due to neurological problem

Give 3 investigations you would consider.


Ankle-Brachial Pressure Index (with or without exercise)
Toe Pressures
Arterial Duplex
CT or MR or DS Angiography

109

Question 91: Cardiovascular


Mrs S.T. is a 75 year old lady. She is overweight and smokes 15 cigarettes a day,
although she is trying to cut down. She has no history of diabetes mellitus and a
recent urine test was normal. She presents now with a 3- month history of pain in her
legs but there is no leg ulceration. You consider that she may have arterial vascular
disease.

List 3 features, on further questioning, which would support a diagnosis of


claudication
The leg pain is located in the thigh/buttock/calf
The pain is exacerbated by exercise and impairs her walking
The pain can be relieved by rest

List 3 features that you would look for on local examination of the leg to support your
hypothesis of chronic arterial vascular disease.
Diminished peripheral pulses
Cold/shiny/scaly skin
Pallor of peripheries
Peripheral hair loss
Ulceration/gangrene
Thickened toenails
Muscle atrophy

She asks you about her condition. In <50 words describe the likely cause of this
presentation as you would outline it to her.
As we get older, the blood vessels that supply oxygen to our muscles may get narrower
Smoking and being overweight are both known to exacerbate this problem
Once severe enough, narrowing prevents the muscles getting enough oxygen, which is felt
as pain

You suspect critical ischaemia. List 4 features which, on examination of the leg, would
support your assessment and suggest that the ischaemia is critical.
Buerger's angle <20 (Elevation Pallor)
Capillary Refill >15 seconds
Diminished/Absent pulses
Evidence of gangrene or ulceration
NOT rest pain as not found on Ex
NOT night pain by hanging off bed
NOT 6 Ps as this is ACUTE Ischaemia - Pain, Paralysis, Paraesthesia, Pallor, Perishingly
Cold, Pulseless

110

Question 92: Paediatrics & Orthopaedics/MSK


A 12 year old boy was referred to the Orthopaedic Clinic by his GP. He noticed a small
swelling on the lateral aspect of his left knee following a twisting injury while playing
football about one month ago. The swelling has since then rapidly increased in size.
He has rest pain, difficulty in weight bearing and limp while walking. There is no
history of fever. A malignant bone tumour is suspected.

What 2 initial investigations will you carry out in the clinic?


Clinical examination - locomotor & neurological of lower limb
L knee Xray - detect tumor

If this is a malignant bone tumour, what appearances would you expect to see in the xray of left knee?
Codmans Triangle - as tumors enlarges raising the periosteum
sunray calcification - new bone breaches the cortex & radiates outwards into adjacent soft
tissue

Name 2 investigations which may be useful in the staging of the disease?


CT/MRI chest abdo pelvis legs - presence of metastases (especially lung)
Lymph Node biopsy - any lymph node involvement

How will you confirm your diagnosis?


Biopsy FNA/core - histological grading/ type

Name 2 treatment options available considering it is only a localised pathology.


surgical resection with margins of exclusion (amputation/limb sparing)
preoperative chemotherapy eg methotrexate with assessment of effectiveness postoperative

What is the prognosis of this condition with available modern treatment methods?
5yr survival 55%

111

Question 93: Psychiatry


You are working in a general practice attachment when Mrs D, a 63 year old woman
whose husband died a year ago, presents to you with a history of feeling tired and
unmotivated for several weeks. Her daughter had contacted you earlier and had
expressed concern that Mrs D had not been looking after herself properly for a long
time. She also stated that her mother had been looking generally unwell. Mrs D is a
smoker and has been hypertensive for 15 years but has no other past medical history

You consider that Mrs D might be depressed. Suggest 2 alternative possible


diagnoses that could account for this presentation.
Grief Reaction
Schizophrenia (-ve symptoms)
Dementia
Substance Abuse
Dysthymia
Organic Disorder e.g. Hypothyroidism

After further examination you decide she is clinically depressed. List 4 mental state
findings that would be sufficient to reach this diagnosis.
Anhedonia
Anergia
Persistently Low Mood
Nihilism
Alexithymia (loss of ability to feel emotion)
Suicidal Thoughts

What is the single most important aspect of risk assessment to consider first?
Suicide Risk/Risk to Others
This is a bad question and weve been told that the author of psych questions has been
informed that it should be removed from further papers.

You decide to prescribe an antidepressant drug. List 3 factors, which would contribute
to your choice of drug.
Suicide Risk (i.e. no TCAs)
Co-morbidities
Drug Allergies
Previous response to antidepressant therapy (if applicable)
Existing therapy (if applicable)
Interactions with current medications (if applicable)

Which other health care professional would you ask to help with longer term follow
up?
Community Psychiatric Nurse
Psychologist
Her own GP

112

Question 94: Obstetrics/ Gynaecology


A 32-year old para 0 + 0 reports regular but heavy and painful periods over the last two
years. She and her husband have used no contraception during this time, in the hope
of starting a family, but pregnancy has not occurred and intercourse has started to be
painful. Pelvic examination reveals a fixed, retroverted uterus and the suspicion of
bilateral adnexal cysts.

What are the 2 most likely diagnoses?


serous or mucinous cystadenoma
ovarian endometrioma

Which 3 investigations would aid the diagnosis?


Transvaginal Ultrasound - detection of any masses or abnormalities
FNA & cytology - confirmation of benign nature of cyst
FBC - WCC + CRP indicate infectious process

What other 3 investigations would you initiate for the couples infertility?
Sperm Analysis - presence of azoospermia, morphological abnormalities
assessment of ovulation/ mid luteal progesterone
hysterosalpingography/laparoscopy

Laparoscopy reveals bilateral ovarian cysts, one of which accidentally ruptures to


release chocolate-coloured fluid. How could you treat this condition medically in the
first instance?
laparoscopic excision of cyst wall, drainage & coagulation of cyst bed

Laparoscopy also reveals damaged tubes, although the other subfertility


investigations prove normal. What assisted conception might be offered to the couple
to help in their quest for a family?
IVF

113

Question 95: Psychiatry


Following a drink-driving charge, a 45 year old sales representative visits your surgery
requesting help for his drinking.

List three features from different aspects of the patients social history which would
indicate the severity of his alcohol problem.
Quantity how much is he drinking each week?
Occupation has his drinking affected his job?
Family/Friends does he remain in contact with them/do his friends drink
similarly?
OR CAGE
Has he ever thought of Cutting down?
Has he ever been Annoyed by people criticizing your drinking?
Does he ever feel Guilty about drinking?
Is alcohol an Eye opener for you?

Name 2 laboratory investigations which would help confirm excessive alcohol use.
LFTs - GGT, AST>ALT
FBC Anaemia or Pancytopaenia
Peripheral blood film Macrocytosis

Give two features of the history you would seek from the patient to indicate he has
alcohol dependency.
According to ICD-10:
Compulsion: Do you feel a strong desire to drink when you dont?
Loss of control: Do you find yourself unable to control your drinking behaviour?
Withdrawal: Do you experience any alcohol withdrawal e.g. shakes, tremor, tachycardia,
palpitations
Loss of tolerance: Do you find yourself drinking more alcohol to reach the same effect as
before?
Salience: Is drinking becoming the most important aspect of your life?
Persistence despite evidence of harm: Do you still continue to drink despite knowing the
harm it does?
Stereotyped repetitive behaviour: Do you find yourself drinking at a certain time each day?
Reinstatement: Have you stopped drinking for a certain period, but then gone on to drink
again?

114

Give four long-term psychiatric complications resulting from long term alcohol abuse.
Neurological:
Wernickes encephalopathy (due to B1 (thiamine) deficiency),
Korsakoffs psychosis (irreversible result of untreated Wernickes
Peripheral Neuropathy
Others = Alcohol-related Dementia, Cerebellar Atrophy, Central Pontine Myelinolysis
Psychiatric:
Anxiety
Depression
Others = Personality Disorder, Amnesia, Alcoholic Hallucinosis, Morbid Jealousy

115

Question 96: Psychiatry


A 23 year old student has recently received a diagnosis of schizophrenia. She has
been prescribed haloperidol. Three days after commencing it she has complaints of
pain in her neck and is noticed to have her face twisted to one side.

What medication side-effect is she suffering from?


acute dystonia

What is the underlying mechanism of action that causes this side effect and in which
brain region does it occur?
D2 receptor antagonism
The Nigrostriatal dopamine pathway in the brain

List 3 other types of side-effect caused by the same drug action.


Tardive Dyskinesia
Akathisa (restlessness)
Parkinsonism

Name 2 management strategies to counteract these side effects


Stop Haloperidol and switch to an atypical antipsychotic e.g. Aripiprazole
Lower the dose of Haloperidol
You would NOT add an Antimuscarinic in practice

Name 2 antipsychotic drugs least likely to cause these side effects


any atypical antipsychotics
Risperido
Amisulpride
Aripiprazole
Clozapine
Olanzapine
Quetiapine

116

Question 97: Obstetrics/ Gynaecology


A 36 year old primigravida is admitted with a blood pressure of
160/100mmHg at 62 weeks gestation. Her legacy has been complicated
until 2 days ago. She attended her midwife today and was referred to
the hospital on account of her blood pressure.

What is the most important diagnosis to consider?


pre-eclampsia

List 4 important symptoms you must enquire about.


Headache
Visual disturbance
Discomfort in upper abdomen
Peripheral oedema

Give 2 pieces of information which will be relevant from her booking attendance.
Baseline BP
FH pre-eclampsia
Dipstick Result
PMH of hypertension

What single besides (or ward) investigation would you arrange to be performed?
Urine proteinuria

Give 3 blood investigation that would indicate disease activity.


Haematocrit
Platelets
Urate
LFTs
The blood pressure to 160/110 over the next 24hrs and biochemical
abnormalities persist.

What is the best treatment for this condition?


Delivery of placenta and baby

117

Question 98: Paediatrics


A 2 month old baby is brought to A&E by his parents with a history that he rolled off
the couch On examination he is conscious but irritable and haas old hands and feet.
His temperature is 37*C and he has no neck stiffness. He has a bgy swelling o the
right side of his scalp, some small bruises his chest and extensive nappy rash. He is
the first child of young unmarried parents.

What is the most likely cause of skull swelling?


skull fracture

What is the most likely underlying mechanism for this?


Trauma (Non-accidental Injury i.e. Shaking Baby Syndrome)

Name at least 2 immediate investigations that should be undertaken to identify the


cause?
Bloods FBC (subdural bleed Hct/WCC), Coag Screen (may be underlying
bleeding disorder)
LFTs for assessment of NAI
Imaging Cranial CT, Cranial US, Cranial MRI, Skeletal Survey (NAI)

The parents are keen to take the baby home as he is now better. As the FY1 in A+E
what would you do?
Seek senior advice immediately
Inform parents that due to the nature of the injury you need to ask a senior colleague to
review child
When speaking to senior colleague you would inform them of your suspicion and ask them to
review

The opthomologist is used to see the child. Name at least one features the are looking
for.
Retinal Haemorrhages
Both Vitreal Haemorrhages and Retinoschisis (splitting of retinas neurosensory layers) may
also be present but are not as likely and so wouldnt give you the marks.

What is the key feature in the history that raises concern?


The child is 2mo old and would have been physically unable to roll off the couch

Name 3 other professionals or resources outside the hospital that might provide
important information to elucidate the diagnosis.
Social Workers
Health Visitor
GP

118

Question 99: Orthopaedics/ MSK


A 25 year old man who weighs 70 kg fell while playing football and sustained an injury
to his right ankle. He was stretchered off the pitch and is now in the resuscitation
room complaining of severe pain. There is an obvious deformity of the ankle joint and
although the skin appears to be intact it is under some tension. You are working in the
Emergency Department where you examine him, and are happy with his airway,
breathing and circulation. His ankle injury is his only problem. This man is screaming
in agony from his ankle injury.

What would you give him for pain relief? Specify the type of drug and give one
example.
morphine sulphate
opioid

What route of administration would you use which would be most appropriate?
IV
He is still in pain. The nurse asks if he may have some Entonox.

What is Entonox?
50% O2 & 50% NO anaesthetic gas
analgesic (equivalent of 15mg morphine given SC)

List 2 groups of patients who should not be given Entonox in an emergency setting.
consciously impaired
patients with a confirmed/ presumed pneumothorax
His pain is now slightly better. You now notice that it is difficult to palpate a pulse in
the right ankle.

What is the most appropriate investigation?


XR of ankle with an AP and lateral view

What additional measures would make him more comfortable before he goes through
this?
Attempt to reduce the fracture and immobilise it in a back slab plaster cast.

If the X-ray demonstrates a fracture/dislocation of the ankle, how should this be


treated?
Attempt to reduce the fracture in a short cast.
If this cannot be achieved then open reduction internal fixation is indicated.

He is immobilised in plaster for 6 weeks. List 3 possible complications that are likely
in this time.
Compartment syndrome.
DVT/PTE
Muscle atrophy

119

Question 100: Paediatrics & Orthopaedics/MSK


A seven year old boy is admitted as an emergency with pain and swelling of his knee.

List 3 likely non-traumatic causes of these symptoms.


Inflammatory Juvenile Idiopathic Arthritis, Bursitis (e.g. Superficial), HenochShonlein Purpura
Infectious Septic Arthritis, Reactive Arthritis, Osteomyelitis
Malignancy Osteosarcoma, Primary Bone Tumour however malignancy is not
a LIKELY cause
NOT Transient Synovitis as not common because knee has capacity to
accommodate fluid

The child refuses to talk to you and clings to his mother. Describe 2 ways to assess
pain in this child.
Ask the child or the mother
Observation of the position he holds his knee in i.e. query pseudoparalysis
General behaviour of child
Observation of gait
Gentle examination if appropriate
Because we put so much research into this and the paediatrician gave us the thumbs up, Im
leaving these in even though they arent on the mark scheme:
Faces Pain Scale child points to face that best represents how they feel
FLACC ranks 5 observational criteria (Face, Legs, Activity, Cry, Consolability)
from 0-2 to determine severity of pain in children aged 2mo-7yrs
Alder Hey Triage Pain Score observational scale similar to above but different
criteria (Cry/Voice, Facial Expression, Posture, Movement, Colour)
Ask mother to play with him/ get him to walk while you observe through a window from a
distance

List 3 features you would look for on examination of his knee.


Joint tenderness
Restriction of movement
Very warm joint / redness
Swelling or deformity

You consider aspiration of the joint. List 2 other investigations you would want to
undertake.
Bloods Culture, FBC ( WCC), cRP, ESR
Imaging US (to guide aspiration) and Plain XR (not diagnostic but used as
baseline)

120

Question 101: Ophthalmology/ ENT


While on an attachment in General Practice you are called to see Mrs Jones who is a
60 year old woman who complains of the sudden development of a painful red right
eye. She has had no previous episodes of a similar nature. There is no history of a
foreign body or trauma.

What 4 questions would you ask, on subsequent history, to help establish a


differential diagnosis?
Are there any visual changes? (e.g. haloes, decreased visual acuity)
What is the discharge like? (none, sticky, purulent, watery, white/stringy)
What does the pain feel like? (e.g. gritty or severe)
Any photophobia? (Uveitis/Corneal Ulcer)
Any nausea or vomiting? (Glaucoma)
Any recent eye drops to examine the retina? (Glaucoma)
NOT any discharge? As everyone with a red eye will have discharge of some sort.
NOT PMH Although PMH important in uveitis for example. Specifics include
inflammatory disease, herpes, allergies, vasculitis, contact lens etc.
You proceed to examine the eye. After inspection of the eye and examination of the
pupil you proceed to evaluate her vision.

What 2 tests will you use and briefly describe how you will carry out each test?
Visual Acuity Chart 6m distance read off letter rows until cannot read next one
Confrontation Test/ Visual Fields sit opposite patient and ask to cover one eye
(while you cover your own) and use red hat-pin to determine their visual fields in
each eye independently.
Though the second point is most likely the correct answer, it would not be the real priority as
field loss from glaucoma is unlikely to be detected in this way. In an elderly patient with a
hazy cornea and loss of vision, you should always stain with fluorescein to rule out a corneal
ulcer

On examination you find evidence of corneal oedema, a fixed dilated vertically oval
pupil, the lack of a red reflex and a shallow anterior chamber. What is the most likely
diagnosis?
Acute Closed-Angle Glaucoma

This patient had attended a diabetic clinic that day. What iatrogenic factor could have
contributed to her acute illness?
Dilating drops/ mydriatics (e.g. Anticholinergics: tropicamide) to help examine retina

What should your next step be?


121

Immediate referral to ophthalmological A&E with analgesia


Management of acute closed-angle glaucoma would involve the following:
The patient is asked to lie supine
Administration of a miotic agent (pilocarpine)
Carbonic anhydrase inhibitor (acetazolamide IV)
Additional agents include: topical steroids, beta blocker (levobunolol. CI: obstructive airways
disease), alpha-adrenergic agonist (apraclonidine).
Analgesia and antiemtics as required and rechecking of the IOP every 30 minutes.

122

Question 102: Psychiatry & Emergency Medicine


A 56-year old man is brought to A&E following an overdose of Amitriptyline.

What is the most concerning toxic effect of Amitriptyline and how should this be
assessed?
Toxic Effect Arrhythmia secondary to QT Interval prolongation
Assessed ECG
After appropriate medical treatment, you interview him with a view to assessing his
mental state.

Give 4 findings in the history which would be suggestive of serious suicidal intent.
Leaving a note
Planning
Undertaking when unlikely to be discovered/interrupted
Did not inform anyone
Patient sure that their actions would result in death
Patient truly wanted to die
Patient would have died without medical intervention
Patient upset that they failed in their attempt
On-going Suicidal Intent

What is the most important aspect of risk assessment to consider?


On-going suicidal intent

List 3 underlying psychiatric disorders most associated with raised suicide risk.
Substance Abuse
Bipolar Affective Disorder
Schizophrenia
Depression
Substance abuse although it has the strongest association with suicide risk, the
consultant felt that it might not be accepted.

123

Question 103: Psychiatry


A concerned mother brings her 21-year-old daughter, Julie, to her General Practitioner.
Since returning from university for the holidays, Julie has been behaving oddly. She
stays in her room all the time and is constantly mumbling to herself. She believes that
she is being chased by a government security agency and that they can hear her
thoughts.

List 4 of Schneiders first rank symptoms of schizophrenia.


(NOTE: different to ICD-10)
Auditory Hallucinations:
3rd Person or a running commentary regarding the subject)
Two or more voices discussing or arguing about the patient
Hearing own thought aloud i.e. thought echo
Delusional Perception
Thought Insertion /Withdrawal/ Broadcast
Made feelings, actions or somatic passivity (delusions of external control)

What may Julie have been doing at university, which is a known precipitating factor
for schizophrenia?
marijuana

Name 2 of the 3 dopamine pathways in the brain. 2 Julie is prescribed a conventional


(typical) neuroleptic drug. Three months later she returns with her mother, who is
upset that she has developed a tremor.
Nigrostriatal and Tuberoinfundibular
Wed just like to point out that there are more than 3 dopamine pathways in the brain.
Mesocortical and Mesolimbic are both also answers bringing the total to FOUR.

Why is this? Name 2 other signs you may see because of this side-effect.
Bradykinesia
Rigidity
This is an ambiguous question. The side-effect they mean is Parkinsonism (not general D2
blockage). Hence, Tardive Dyskinesia and Acute Dystonia are incorrect.
Julies mother attends the surgery alone to ask about the treatment and what the
prognosis is for her getting back to University.

Explain in less than 40 words what you would tell them.


As Julie has capacity and has capacity you are legally obligated to follow her wishes
Julie has not made it clear that her mother can be informed about specifics of her rx
You are unable to discuss Julies case with her without Julie present and consent being given
HOWEVER, as a general rule of thumb, 1/3 recover, 1/3 will relapse and 1/3 will never
recover

124

You refer Julie to a psychiatrist. After 1 month she has failed to respond to two
neuroleptics and clozapine is prescribed. He explains to you that Julie should now
attend the GP surgery for regular blood tests as part of routine monitoring.

What test should be done and what abnormality is being looked for?
agranulocytosis

125

Question 104: Paediatrics & Gastrointestinal/Hepatic


A five week old boy is brought for the second time in one week to Accident &
Emergency with persistent vomiting. The parents say that his vomiting started 10 days
earlier and that after vomiting he is always hungry. At the last visit he was diagnosed
as having gastro-oesophageal reflux and started on feed thickeners and Gaviscon.
Until the age of one month he was well and had been thriving.

You are working as a Foundation Training Doctor in A&E and examine the child and
perform an arterial blood gas which is reported as follows:
PH 7.50 (normal range 7.35-7.45);
pCO2 5.0 kPa (normal range 4.6-6 kPa);
p O2 12 kPa (normal range 10-13.5kPa);
base deficit +5.8 (normal = 0)

How would you assess his degree of hydration?


History - previous limited intake, thirst, abnormal losses, comorbidities
Examination - pulse, BP, CRT, JVP edema (peripheral, pulmonary), postural hypotension,
GCS/degree of consciousness- restlessness/irritability associated with mild loss, severe loss
associated with reduced conscious level
Clinical monitoring - NEWS, fluid balance charts, weight urine output
Laboratory assessments - FBC, Urea, Creatinine, electrolytes

List 5 indicators of dehydration.


HR, SoB, CRT >2s
absence of tears
sunken fontanelle
sunken eyes
tenting
sunken abdomen
dry mucous membranes
poor skin turgor

Describe the abnormalities seen in the blood gas results.


pH - alkalosis
base deficit - anion gap
metabolic alkalosis

Please describe, in <50 words, how these changes arise.


1. (chloride depletion; 1* factor) loss of Cl- -> HCO3- reabsorption in the kidney
(= pH) to maintain electrical neutrality by compensation for lost Cl-.
2. loss of H+ -> HCO3- produced in gastric parietal cells is maintained at higher
proportions to the H+ pH
3. loss of K+ -> shift of H+ into cells & in absence of K+ renal tubule H+
excretion is
4. (contraction) loss of Na+ -> ECF vol promotes aldosterone (via RAAS) in
distal tubules reabsorption of Na+ in exchange for loss of H+

126

On more detailed examination you notice gastric peristalsis and you feel a pyloric
tumour in the upper abdomen. What is the most appropriate next investigation?
abdominal ultrasound

What is the most likely diagnosis given this clinical scenario?


hypertrophic pyloric stenosis

What would be the operative procedure of choice?


pyloromyotomy

127

Question 105: Paediatrics


A mother brings her 15 month old son to see you. She is worried that the child is not
yet walking. He started crawling 2 months ago but cannot yet stand unsupported
without falling.

List 3 possible reasons which might explain this childs motor development.
normally variance
mild cerebral palsy- hemiplegic cerebral palsy
fragile X syn
rickets
duchenne muscular dystrophy
spinal muscular atrophy
global developmental delay of any cause i.e. TORCH exposure in utero, any chromosomal
abnormality

What further points in the history will you seek? List 3


History of pregnancy - any antenatal infections, exposure to prescribed/ recreational drugs,
maternal illness?
Peri-natal history - premature. duration of labour, mode of delivery, birth weight, resus
needed, birth injury, congenital malformations identified?
Any neonatal illness - fits, febrile illness, feeding problems?
Any surgery, accidents, trauma?
Progression in comparison to any siblings?
any notable family history - Fragile X, DMD?
Concerns about other areas of development i.e. fine motor, speech and language , social
and emotional

You decide to evaluate other aspects of the childs development. What would be the
normal speech and language development you would expect in a 15 month old child?
Give 2 points.
vocabulary up to 5 words
adopts no as favorite works
puts fingers to mouth & says shh

You continue with your developmental examination. How would you test vision in this
child? List 2 points
Test visual acuity using Cardiff Cards
if <6wks premature, visual acuity should be 6/18 to 6/12

128

Question 106: Psychiatry


You are asked to see a 22 year old woman who has been brought to the Emergency
Department by her friends. She is agitated, preoccupied and suspicious. Over the past
couple of days she has complained that strangers have been following her and
monitoring her movements. You suspect she may be suffering from schizophrenia.

List 3 possible organic differential diagnoses.


hyperparathyroidism
hyperthyroidism
hepatic encephalopathy
medicine-induced psychosis
heavy metal poisoning (eg bromide, mercury)
carbon monoxide poisoning
epilepsy
brain tumours
traumatic brain injuries
HIV
herpes encephalitis
substance induced

What urine investigation would be appropriate in any person presenting in these


circumstances?
urine drug screen - may identify causative drug if substance induced

List 4 symptoms regarded as characteristic (first rank) of schizophrenia


auditory hallucinations:
hearing thoughts spoken aloud
hearing voices referring to himself / herself, made in the third person
auditory hallucinations in the form of a commentary
thought withdrawal, insertion and interruption
thought broadcasting
somatic hallucinations
delusional perception
feelings or actions experienced as made or influenced by external agents

List 2 findings on mental state examination that would make a diagnosis of


schizophrenia less likely.
Appearance - rash around mouth (inhalant abuse), needle track marks, nicotine stains,
dental erosion
delusions are not as crystallized as in schizophrenia but auditory hallucinations may still be
present.
drug history may reveal abuse of eg cannabis known to be associated with psychosis in
chronic abuse

129

Question 107: Paediatrics


An infant of 7 months suddenly became irritable. His mother noticed that he was
drawing his legs up when in pain and had passed red stools in his nappy. He was
otherwise well but had episodes of intermittent colic over the preceding 24 hours. You
suspect that he may have an intussusceptions.

Describe, in one sentence, the abnormality that occurs in intussusception


Telescoping (invagination) of one part of the intestine into another

List 3 physical signs that may be present


Palpable sausage-shaped mass typically in RUQ or epigastric region
Blood on PR exam
Positive Dances Sign retraction of right iliac fossa
Signs of dehydration e.g. skin turgor, dry mucous membranes

What 2 investigations could you perform to help establish the diagnosis?


US target lesion/donut sign
AXR might show edge of intussusception

List 3 complications of this condition


Intestinal Perforation and/or Ischaemia and/or Necrosis
Hypovolaemic Shock
Sepsis

How can this condition be treated? Give one example


Fluid Resuscitation + Air or Contract Enema Reduction (if no contraindications)
OR
Fluid Resuscitation + Surgical Reduction +/- broad-spectrum antibx

Other than intussusception name 2 other conditions that could account for his
symptoms.
gastroenteritis due to Salmonella or Shigella
Urinary tract infection (typically urinary symptoms more prevalent)
pyloric stenosis (but projectile nonbilious vomiting postprandial)
appendicitis (but PR bleed not usually present)

When taking the history from the mother, name 2 symptoms that it is important to ask
about.
blood/mucus or Redcurrant jelly stools (late sign)
Episodes of pallor and screaming
Vomiting (Bilious - late sign)
On examination, the child becomes increasingly irritable. He becomes pale, develops
a tachycardia of 140 bpm and a capillary refill time of 5 seconds. His respiratory rate is
30 per minute but saturations are 98% on air.

Name 3 steps that you would perform in his initial management.


130

IV fluid resuscitation 20ml/Kg normal saline boluses


IV access & NBM
analgesia
Notify ED/team Consultant & surgical registrar
AXR - exclude perforation or bowel obstruction

What investigation would confirm a diagnosis of intussusception?


Ultrasound Abdomen

What classic sign may it show?


target/ doughnut sign - concentric alternating echogenic/ hypoechoic bands with the
echogenic bands formed by mucosa & muscularis and the submucosa forming the
hypoechoic bands.
pseudokidney - longitudinal appearance of intussuscepted segment of bowel
crescent in a doughnut sign - crescent is formed by mesentery dragged into the
intussusception

131

Question 108: Paediatrics


It is 11.30pm. You are working as a paediatric FY1 when a midwife asks you to see a
baby boy who is three hours old, having been born at term. The midwife thinks the
child may have Downs syndrome. You examine the infant and you are also certain
that the infant does indeed have Downs syndrome.

Name 6 clinical features you might see on simple visual inspection which are
consistent with a diagnosis of Downs syndrome.
brachycephaly + flat occiput
epicanthal folds
upslanting palpebral folds
Brushfield spots on riris
short nose with low nsala bridges and small nares
excess skin on back of neck
short hands
single palmar transverse crease
digital dermatoglyphics
5th finger clinodactyly
wide space between 1st/2nd toes
vertical plantar creases
hypoplasia of iliac wings

What neurological finding is present in almost all newborn infants with Downs
syndrome?
Hypotonia

You decide to confirm the diagnosis. What is the most likely karyotype?
Trisomy 21

Name 4 gastrointestinal anomalies are associated with Trisomy 21.


duodenal/ anal stenosis
duodenal/ anal atresia
Hirschsprung disease (2-15%)
tracheoesophageal fistula (1%)

132

What type of karyotype would significantly increase the risk of his parents having
another baby with Downs syndrome?
t(14q:21q) - Robertsonian Translocation
Caused by translocation between the long arms (q) of chromosomes 14 and 21 where the
breakpoint is near the centromere; the short arms (p) of these two chromosomes are
generally lost. Carriers are clinically normally but outcomes from conception can include T21
(live birth), T14 (early miscarriage) or t(14q21q) carrier (live birth). - High-yield Cell and
Molecular Biology, Volume 845 p60 (on google books).
A women carrying silent RT 14q;21q has high risk (12-15%) for having a child with Down
syndrome. - Kolgeci S, Kolgeci J, Azemi M, et al. Dermatoglyphics and Reproductive Risk in
a Family with Robertsonian Translocation 14q;21q. Acta Informatica Medica. 2015;23(3):178183. doi:10.5455/aim.2015.23.179-183.
PALI have this answer as being 21:21 but after discussing with Sophie we would disagree
and state the answer above. Ive added the text above to support out answer.
Examination of the cardiovascular system at the time of birth was normal. Five weeks
later the mother takes the baby, James, to the General Practitioner because he has not
been feeding well. On examination the General Practitioner hears a loud pansystolic
murmur but noticed that James is not cyanosed and all his peripheral pulses are
palpable.

Give 2 cardiac lesions that are most likely to underlie his signs and symptoms?
Atrioventricular Septal Defect
Ventricular Septal Defect

The babys mother is in the room. How should the diagnosis be given? Give 5 points.
By a senior clinician with a nurse present to support the mother
In a quiet/calm area suited to discussion (i.e. not in a ward with other people)
Use of warning shot without use of directly negative statement e.g. Im afraid its not good
news
Over an adequate amount of time to facilitate discussion i.e. do not rush
With a detailed description of the condition and the impact it will have (if desired by the
mother)

133

Question 109: Emergency Medicine


A 24-year-old man has been brought to A&E department following a motorbike
accident. He is tachycardic and hypotensive (pulse 120 per min blood pressure 80/ 40
mmHg). Clinical examination shows bruising over left side of pelvis and tenderness on
pelvic compression test. There is blood at the external urethral meatus. You suspect a
fractured pelvis.

Other than fracture, name 2 other associated injuries which could cause hypotension
in this case?
internal haemorrhage eg left thigh - haemothorax, splenic rupture
neurological -> head injury or C-spine injury (loss of sympathetic supply)

What is the significance of blood at the external urethral meatus and what clinical
examination will you perform to confirm your suspicion?
urethral injury
PR - boggy riding prostate

Give 2 examples of X-rays that must be carried out during evaluation of a polytrauma
patient.
Chest X-ray AP
Pelvic X-ray AP

Accepting that there are no other associated injuries, describe 2 forms of treatment
that you would use to treat the uncontrolled haemorrhage in this patient?
Pelvic Binder (stabilises pelvis stasis)
Fluids O- blood or cross matched or fluids
Clotting Factors FFP cryoprecipitates, Octoplex
Other US/ Laparotomy/ Packing/ Angiogenic Embolisation/ Ligation

What is the overall mortality rate associated with this injury?


10-30%

Name 2 causes of death.


pulmonary embolism
infection
shock (hypovolemia)

134

Question 110: Paediatrics & Obstetrics/ Gynecology


You are called to the postnatal ward to see a well-looking baby who is 16 hours old.
She was born at term following a normal labour and vaginal delivery. She has had
three good breast feeds since birth and is well hydrated. However, the mother and
midwife have noticed that the baby is quite jaundiced. The mother is Asian and her
blood group is A Rhesus negative. She has one previous normal child who was born
in rural India.

What is the most likely diagnosis in this case?


Haemolytic Jaundice

List 4 investigations which should be performed to investigate jaundice in a neonate.


Establish Diagnosis:
Transcutaneous Bilirubinometer
Split Bilirubin Test

Identify Cause:
Direct Coombs test
Peripheral Blood Smear, Group + Save
G6PD test, FBC, Abd US, Liver biopsy etc.

You decide to start treatment pending the results from your investigations. What
treatment would you start?
Phototherapy

List 2 possible complications of this treatment.


Loose Stools
Skin Rash
Potential Retinal Damage
Insensible Water Loss

What other more invasive treatment option is available if your first-line therapy fails?
Exchange transfusion

What is the most important consequence of failing to adequately treat the jaundice?
Kernicterus (necrosis of neurons in brainstem, hippocampus and cerebellum
paralysis of upward gaze, sensorineural hearing loss, dental dysplasia,
intellectual deficit and chorio-athetoid cerebral palsy)

135

Question 111: Paediatrics


A 6 year old boy is brought to casualty with a four hour history of abdominal pain and
of feeling hot. He has pain in the right iliac fossa with rebound tenderness and a
temperature of 37.3*C.

List 3 potential causes for these symptoms and underline the most likely diagnosis.
appendicitis
urinary tract infection
mesenteric adenitis
infectious gastroenteritis

Give 2 features which, on further questioning, would support this diagnosis.


pain that started periumbilically and migrated to right iliac fossa
movement exacerbates the pain
loss of appetite
nausea/ vomiting
(occasionally) loss stools

Apart from those above, give 2 features which, on examination, would support this
diagnosis.
tachycardia
Rovsing sign - RIF pain when palpating LIF
Psoas sign - RIF pain on hyperextension of R hip
Obturator sign - RIF on internal rotation of flexed right hip
McBurney's sign - RIF over appendix

The child suddenly develops widespread severe abdominal pain and becomes
shocked. The blood pressure is now 70/40. What is the most likely cause of such a
development?
perforated/ ruptured appendicitis with generalised peritonitis secondary to ruptured appendix

Give 2 findings on examination or investigation which would support this diagnosis.


FBC - WCC
USS - RIF abscess/ phlegmon + signs of appendiceal inflammation/ appendicolith (eg
aperistalitic/ non-compressible structure with outer diameter >6m)
urinalysis - negative
Erect CXR - air under diaphragm
abdominal/ pelvic CT scan - abnormal appendix/ diameter >6mm, calcified appendicolith,
periappendiceal inflammation
Examination; reflex guarding and progression to rigidity, percussion pain, absent bowel
sounds

Give 2 therapeutic measures which might subsequently be performed in this child.


appendectomy ?laparotomy if generalised peritonitis
IV antibiotics (eg gentamicin & metronidazole) + supportive care
CT guided/ operative drainage of abscess
136

137

Question 112: Obstetrics/ Gynecology


A 24-year old primigravida presents at thirty-two weeks in a previously uneventful
pregnancy. She is symptom-free apart from marked facial oedema, but her blood
pressure is sustained at 145/105 and there is proteinuria (+) on testing. You arrange
her admission for further investigation and management.

List 4 investigations that would help you assess the maternal condition.
FBC Platelets (?decreased, which may suggest HELLP)
LFTs Transaminases (part of HELLP)
U&Es Creatinine (elevated -> underlying renal disease or (rarely) development
of failure)
Urate
Abdominal examination shows a fundal height of 26cm with apparently reduced liquor
volume.

List 3 ways ultrasound can be used to help assess the fetal condition.
Estimated fetal weight
Biophysical profile
USS Doppler of Umbilical Artery (for waveform)
Check movements of foetus
Liquor volume is another answer here, but given that it is mentioned directly before this were
not certain it counts as an answer - particularly when there are other options available. Also,
note that although Biophysical Profile is on the mark scheme apparently but O&G dont
actually do it anymore.

What other investigation would help reassure you about fetal well being?
CTG (Cardiotocography) measure variability of heart rate
Delivery of the baby by caesarean section is planned, in the foetal and maternal
interest.

How can the administration of steroids help the survival of the preterm infant?
accelerates fetal lung maturation and increases surfactant synthesis and reduces the risk of
respiratory distress syndrome

What is the most likely diagnosis in this mothers instance?


pre-eclampsia
Delivery of the baby by caesarean section is planned, in the foetal and maternal
interest.

How can the administration of steroids help the survival of the pre-term infant?
Reduces the risk of neonate developing RDS by stimulating production of pulmonary
surfactant

138

Question 113: Emergency Medicine & Paediatrics


You are asked to see a 1 year old girl in A&E with a generalised convulsion. She has
been brought in by ambulance 2 minutes earlier, although the fit has been continuing
for about 10 minutes. She had been unwell during the previous 24 hours and was
noted by her mother to have a temperature of 40C shortly before the fit commenced.

Give 4 appropriate actions you would perform immediately, before giving specific
medication.
Call for senior help
A protect the airway
B High-flow oxygen through trauma mask + pulse oximetry + auscultate for
breath sounds
C large bore IV access, fluid resuscitation, ABG, Bloods (FBC, U+E, LFTs)
D conscious level, check pupils
E check for rash (e.g. petechial non-blanching) and re-assess temperature
BM
Though Insertion of a Urinary Catheter is a valid component of C, it should always be done
after seizure cessation

You decide to stop the fit. How can this most appropriately be done?
Benzodiazepines e.g. IV or PR Diazepam or buccal Midazolam or IV Lorazepam
PR paraldehyde would be the next step after benzos
If this didnt work then escalate to PR paraldehyde
If this still doesnt work then IV AEDs e.g. Phenytoin
If the patient is still seizing post-AEDs, then contact ITU urgently as GA may be required for
cessation

What is the most likely diagnosis?


Febrile convulsion

After successfully completing your initial treatment you decide to take steps to control
the childs temperature. Suggest 3 means of achieving this.
Drugs NSAIDs, Paracetamol. Aspirin,
Stripping the child
Removing Blankets
There is no real evidence to show that wet towels or fan work although they are reasonable
suggestions.

The childs mother later asks you if this episode means that her daughter should not
have her next scheduled immunisations. Is she eligible to have the immunisations
under these circumstances?
Yes unless they are febrile or unwell at the time of vaccination
Half an hour after termination of the fit, the childs condition has improved
considerably. You examine her and find that she is miserable but alert with no signs of
meningitis and nothing to suggest a focus for infection.

What 1 investigation would you consider essential in this girl?


139

Urinalysis/Urine Culture to ensure UTI is not missed

When the child goes home what 2 pieces of advice would you give to the parents
about preventing another febrile convulsion?
Ensure they have a thermometer so they can closely monitor childs temp when they are
unwell
Ensure they have a supply of paracetamol and ibuprofen in the home to give if child is hot

What is the prognosis for this girl concerning liability to further convulsions?
~33% of patients will have another febrile seizure, while 10-20% will have 3
further episodes with most occurring within 2 years of the initial presentation
There is also a 2-5% risk of developing Epilepsy depending on the type of febrile seizure.

140

Question 114: Psychiatry


A 28 year old bank teller visits your GP surgery having received a verbal warning at
his work. He gives a year long history of worries that his receipts will not balance each
day to the extent that he has to check each transaction at least six times. Customers
have complained about his excessive slowness.

What are these symptoms?


compulsions

List 5 characteristic features of such symptoms.


Repetitive behavioural or mental acts
Neutralise anxiety
Not inherently enjoyable
Attempts made to resist typically fail
Self-reinforcing
Patient sees compulsions as a way of preventing an unlikely occurrence
Compulsions are Egodystonic (i.e. the pt. recognises them as unreasonable)

Name 2 possible diagnoses.


OCD (sees obsessions as a bad thing)
OCD PD (sees obsession as a good thing)

What neurotransmitter system is thought to be most relevant in the genesis of these


symptoms?
5HT

What drug treatment is most effective?


TCAs e.g. Clomipramine
SSRIs e.g. Escitalopram
This question is ambiguous due to wording. The most effective treatment is clomipramine,
but its not the first line youd use plus, given that the previous question is asking about
serotonin, we wondered if it was designed as a lead in to us writing SSRIs. Weve been told
if you write a very brief answer explaining that SSRIs are the 1 st line but that Clomipramine
is the most effective then you wont miss out on the marks since youd be correct.

141

Question 115: Paediatrics & Haematology


A mother brings her 3-year-old daughter Rebecca to the GP surgery. Rebecca has
been unusually tired and is now refusing to walk. On examination you note that
Rebecca has several bruises and is very pale.

Define Leukaemia
Haematological Malignancy
Presence of neoplastic haemopoietic cells in the bone marrow +/- peripheral circulation

What is the most common type of leukaemia in children?


ALL - acute lymphoblastic leukemia

The presence of a Philadelphia chromosome in children with the most common form
of leukaemia is only 3%. What is the Philadelphia chromosome?
Translocation of 9 and 22

With which type of adult leukaemia is the Philadelphia chromosome most commonly
associated?
CML - chronic myeloid leukemia

Apart from the Philadelphia chromosome, which other chromosomal abnormality


predisposes children to leukaemia?
Trisomy 21

What 4 investigations should be done to confirm Rebeccas leukaemia, giving the


features you might expect to find on each?
Full Blood Count
Peripheral Blood Film
Bone Marrow Aspirate/Trephine

Rebecca is prescribed allopurinol. For what complication of treatment is this given,


and what is the associated electrolyte abnormality the oncologist is trying to avoid?
To prevent Tumour lysis Syndrome

What electrolyte abnormality is the oncologist trying to avoid by prescribing


allopurinol?
Hyperuricaemia

142

Question 116: Obstetrics/ Gynecology


A primigravida is normotensive at booking at 12 weeks gestation. At 34 weeks
gestation she is found to have a blood pressure 150/90mm Hg. She has proteinuria ++
+ on dipstick testing. You see her as the FY2 in the Day Care Unit.

What important points would you like to elicit from her history? Name 3.
Any symptoms being experienced - headache (frontal), upper abdominal pain, visual
disturbances (eg scotoma, photopsia), LoC/ seizures, oliguria, vomiting, sudden swelling of
hands, face, feet?
Any positive FHx, her age ( RFx if >35yrs), multiples, re-gestational DM, any
comorbidities (eg renal disease, autoimmune disease)?

Apart from blood pressure assessment and urinalysis, name 4 investigations that you
would carry out in a Daycare setting and explain why these tests should be done.
Physical Examination - edema, fundus-pubis symphysis height
Spot urinary protein:creatinine ratio/ 24-hour urine collection - quantify proteinuria.
U&Es + eGFR - assess kidney function
LFTs - excl transaminases in HELLP
FBC - excl plts in HELLP
Fetal USS - assess fetal well being by size of baby and amniotic fluid volume
Fetal CTG - assess fetal well being
Fetal biometry - ?fetal growth restriction
Umbilical a. Doppler velocimetry - assessment of end diastolic flow
?signs of raised ICP; fundoscopy

She remains hypertensive and is found to have 5gms proteinuria /24 hours. What is
your diagnosis?
(Moderate) pre-eclampsia

How might this condition affect the fetus? 1 Suggest one only.
intrauterine growth restriction
stillbirth

Which groups of antihypertensives are most appropriate for use in pregnancy.


Suggest 1
Beta Blockers eg labetalol
Alpha adrenergic antagonist eg methyldopa
Calcium Channel Blocker eg nifedipine

143

Question 117: Obstetrics/ Gynecology


A 40 year old woman recently had a hysterectomy and bilateral oophorectomy for
menorrhagia. She now wishes advice for hormone replacement therapy (HRT).

Give four symptoms which would suggest oestrogen lack.


urogenital symptoms - dyspareunia, vaginal discomfort, dryness, recurrent lower UTIs,
urinary incontinence
loss of libdio
brittle nails, skin thinning, hair loss
vasomotor symptoms - c flushes, night sweats, headache

Give three possible contraindications to HRT which you would look for in this
patients past history.
undiagnosed abnormal vaginal bleeding
venous thromboembolic disease
active/ recent angina or MI
suspected, current or past breast cancer
endometrial cancer or other oestrogen dependent cancer
active liver disease + abnormal LFTs
uncontrolled HTN
Also but not relevant to this pt: pregnancy/ breast feeding

Give three long term advantages of HRT for this patient, apart from relief from
menopausal symptoms.
bone marrow density and prevents osteoporotic fractures
intervertebral disc collagen loss maintain strength & function and prevent
crush fracture
reduces CHD (if started <60yrs)
effective management of depression in postmenopausal women
replaces lost skin and collagen to provide cosmetic effect

Which generic form of HRT would be most suitable for this patient and why?
estrogen alone as progesterone would only be needed if the pt had a uterus (or coexisting
conditions such as endometriosis that required a progesterone counter)

144

Question 118: Obstetrics/ Gynecology


A 27-year old woman in her first pregnancy presents at 32 weeks gestation with a
sudden gush of clear fluid from the vagina. She has no contractions and has had no
show. You suspect she has ruptured her membranes.

List 2 complications of preterm rupture of membranes.


Preterm Labour
Infection Chorioamnionitis
Foetal Distress/Death
Placental Abruption
Cord Prolapse
You arrange for her admission and perform a sterile speculum examination.

What observations would you make and/or what investigations would you perform
during the speculum examination? List 3
Observations:
Visualise Amniotic fluid draining through cervix
Degree of dilation of Cervix
Check for pooling of fluid in post fornix
Look for offensive yellow/brown discharge
Investigations:
High vaginal swab for infective agents (e.g. Chlamydia Trachomatis)
Choriamniocentesis

List 3 variables that you would wish to observe to try and identify the onset of
complications of premature ruptured membranes?
Pre-term Labour dilation of cervix, uterine contractions and foetal heart rate
Chorioamnionitis maternal temperature, WCC/cRP levels (), uterine
tenderness
Foetal Distress on CTG

The baby is delivered. In the first 4 hours after delivery what respiratory signs in the
infant would suggest Respiratory Distress Syndrome? List two.
Cyanosis
Nasal Flaring
Intercostal Indrawing
Subcostal Recession

145

Question 119: Paediatrics & Infection


You are on a paediatric attachment when a 4-month old boy is brought to casualty with
a short history of vomiting and fever. After seeing him you decide that he needs a full
septic screen.

List 2 methods for obtaining an uncontaminated sample of urine from this boy and
explain when and why you might make the different choice of method
Catheter specimen
Supra-pubic aspiration
Clean-catch mid-stream specimen

Name 2 other parts of the septic screen, excluding a full blood count, which you would
perform to identify the infection in this instance.
Culture blood, urine and CSF (if not contraindicated) for culture and sensitivity
Microscopy urine and CSF (if not contraindicated)
Bloods WCC/ ESR / cRP
Imaging CXR
NOT Stool Sample although a part of the sepsis screen, the above are more important

48 hours after admission the microbiologist informs you that the laboratory is in the
process of isolating a bacterium from the urine. What 2 standard criteria are used to
define a laboratory culture diagnosis of urinary tract infection?
Pyuria Positive/Negative
Bacteriuria Positive/Negative
Not in NICE Guidelines but valid Colony Forming Unit:
SPA Sample = >1000cfu/ml
Catheter Sample = >10,000cfu/ml
Clean-catch Sample = >100,000cfu/ml

What is the usual organism (or class of organism) which is obtained from a positive
urine culture in this setting?
E.Coli

The urine infection is confirmed and the infant is treated with intravenous antibiotics
for 5 days, followed by a course of oral antibiotics; he improves steadily over this
time. What treatment should he receive on discharge pending further investigations?
Prophylactic antibiotics

What single initial radiological investigation would you request for him at follow-up?
Within 6wks of UTI an US of the urinary tract should be performed to identify structural
abnormalities.
DMSA (identify renal abnormalities) cannot be performed until 4-6mo following the acute
infection. MCUG should only be performed in atypical or recurrent UTI.

146

Question 120: Paediatrics & Gastrointestinal/ Hepatic


A 6 year old boy is brought to casualty with a four hour history of abdominal pain and
of feeling hot. He has pain in the right iliac fossa with rebound tenderness

What is the most likely diagnosis? (In 2010 asks for 3 ddx + underline most likely)
Acute Appendicitis
Gastroenteritis
Mesenteric Adenitis
Meckels Diverticulitis

Give 2 features which, on further questioning, would support this diagnosis.


Migration of pain from central abdomen to become localised in RIF
Anorexia

Give 2 features on examination that would support this diagnosis.


Rovsings Sign greater pain felt in RIF on palpation of LIF
Tenderness over McBurneys Point

The child suddenly develops widespread severe abdominal pain and becomes
shocked. The blood pressure is now 70/40. What is the most likely cause of such a
development?
Perforated Appendix causing peritonism

Give 2 findings on examination or investigation which would support this diagnosis.


Air under the diaphragm on erect CXR
Rigid, board-like abdomen on palpation

Give 2 therapeutic measures which might subsequently be performed in this child.


IV Antibiotics
Urgent Laparotomy

147

Question 121: Emergency Medicine


A 43 year old fireman is brought in to Accident & Emergency where you are working.
The fireman has burns to his face, neck, scalp, around the wrists and hands.

You are concerned he may also have an airway problem. List 4 symptoms and/or signs
that may indicate an airway problem in this man.
Stridor
Soot in mouth/ sputum/ nares
Dyspnoea
Oro-facial/ tongue swelling
Hoarseness

You tell the nurse with you that you plan to admit the patient. List 3 admission criteria
for burns which may apply to this patient.
Special Site e.g. burns of face/hands
Smoke inhalation injury
IV treatment necessary
Circumferential burns

How can you assess the depth of burns to the skin?


Sensation
Appearance
Capillary Refill
Partial Thickness painful/red/blister Full Thickness painless/white/no
perfusion

You request an opinion from the Plastic Surgery team. They decide an escharotomy is
needed in the affected hand. What is an escharotomy and why is it being done?
Cutting the eschar (thick rigid layer or burnt tissue)
Necessary for full-thickness circumferential burns of neck, thorax and extremities
Performed to prevent compartment syndrome restricting blood flow

148

Question 122: Neurology


You are a House Officer who has been called to the Accident & Emergency
Department. You have been asked to see Mr MacDougall, a 30-year old male who has
been brought in by ambulance with a short history of severe headache of sudden
onset. He was previously well and there is no other significant history. He is now
unconscious and his breathing is impaired.

What scoring system will you use to quantify the level of consciousness?
Glasgow Coma Scale

What important conditions would you suspect to be the cause of the problem? List 3.
Sub-Arachnoid haemorrhage
Pyogenic Bacterial Meningitis
Intracerebral haemorrhage
Encephalitis

What acute complication may be precipitated by a lumbar puncture?


Coning and compression of the brainstem

What is the most important investigation you would arrange to try and confirm the
diagnosis?
CT head
Within 24 hours of presentation, in spite of appropriate management, the patients
general condition deteriorates to a point at which brain stem death is pronounced.
Organ donation is contemplated.

What medical conditions would preclude organ donation? List 2.


HIV positive
Malignancy (Visceral or Haematological)
CJD

The Consultant in charge of the patients case is talking to the family about the
concept of brain stem death. What key points need to be covered? Suggest 2.
There is irreversible damage to the area of the brain responsible for breathing and
maintaining life
The patient is not aware of their surroundings nor are they in any pain
Currently life is being supported by machine

149

Question 123: Neurology & Psychiatry


A 65 year old male arrives in Accident and Emergency, smelling strongly of alcohol,
having been found unconscious after an alleged assault. He is well known to the
department, having last presented 1 week ago with alcohol withdrawal. You check his
airways, breathing and circulation, stabilise his cervical spine and begin clinical
examination.
Routine observations are normal, and examination demonstrates two finger-breadths
of hepatomegaly but no stigmata of chronic liver disease. Routine blood tests show
no differences from results from last week. You decide he should be admitted
overnight. He has no next of kin.

In a Scottish hospital, who can give consent for further medical treatment, and under
what auspices?
Adult with Incapacity Scotland 2000 (Part 5)
If there was a welfare attorney, consent should be gained from them if practical to do so
If not, the doctor is authorised to treat the patient

You decide to manage him as alcohol withdrawal syndrome. List 2 forms of treatment
that you would institute.
Parenteral vitamin supplementation e.g. Pabrinex
Reducing regimen of BDZ e.g. Chlordiazepoxide
You are called to the ward an hour later. The patient is increasingly agitated, and
unresponsive, you demonstrate a newly enlarged pupil on the left side with
corresponding upper motor neurone signs in the left arm and leg. His blood glucose is
in the normal range.

Please give 2 most likely causes of this acute deterioration.


Left-sided Tentorial Herniation due to raised ICP (secondary to intracranial haematoma)
Acute Brainstem Stroke

Please list 4 other causes of reduced Glasgow Coma Scale in such a patient.
Delirium Tremens
Meningitis/ Encephalitis
Hepatic encephalopathy
Seizures
Alcohol Intoxication
BDZ Overdose

What is your next investigation?


CT Head

Whilst you are arranging investigations, nurse tells you the patient is having recurrent
tonic-clonic seizures. You diagnose status epilepticus. Name 2 drugs you could use to
terminate this episode.
Lorazepam
Phenytoin

150

Question 124: Paediatrics & Infection


A child of 5 years presents to her GP with a 3-day history of severe pain swelling and
tenderness in her left forearm, with fever and malaise. Her forearm is swollen and
inflamed. You suspect a possible deep seated infection in the bone.

List 2 investigations that would help you arrive at the diagnosis of osteomyelitis.
Bloods FBC, ESR, cRP
Imaging XR, Bone Scan or MRI of forearm
Cultures blood, of aspiration from joint, from bone following debridement

Name 2 other possible causes of a swelling in the forearm you would include in your
ddx.
Cellulitis
Ewings Sarcoma / Osteosarcoma
Trauma (fracture with possible compartment syndrome)

List 3 important steps in your initial management of this patient and why they are
relevant.
Analgesia e.g. paracetamol to reduce distress,
Arrange admission and referral to orthopaedics for possible surgical drainage if initially
unresponsive
Empirical Antibiotics e.g. Flucloxacillin and Benzylpenicillin to treat infection
and prevent bone necrosis, chronic infection with discharging sinus, limb
deformity and amyloidosis

One treatment is started, how would you monitor the patient to ensure the condition
was improving. Give 3 examples
Monitor vital signs i.e. pyrexia, HR, RR and improvement of symptoms
Bloods repeat looking for reduction of CRP, ESR
Imaging repeat plain film/MRI looking for improvement

151

Question 125: Paediatrics & Opthalmology/ENT


7 year old boy presents to the ENT outpatient clinic with a 6 month history of nasal
blockage, recurrent ear pain and difficulty hearing. He has been previously treated
unsuccessfully with antibiotics and decongestants.

List 2 diagnoses you would consider as a cause for his symptoms.


Acute Otitis Media with Effusion (aka glue ear)
Recurrent Acute Otitis Media
Adenoid Hypertrophy
Foreign Body
Allergic Rhinitis

Suggest 4 further features in the history which would help clarify the diagnostic
possibilities and/or evaluate the symptoms.
Discharge from ear?
Severity of hearing loss assess via impact on school, irritability, speech
problems
History of Atopy
History of Eczema

A pure tone audiogram demonstrates bilateral, low frequency conductive hearing loss.
What surgical treatments may be offered for his hearing loss?
Grommets
Excision of underlying cause if adenoid (adenoidectomy)

Explain how this treatment would improve his symptoms.


Grommet facilitates improved hearing via conduction of sound
Adenoidectomy improved nasal obstruction

152

Question 126: Obstetrics/ Gynaecology


You are attached to a general practice antenatal clinic. You review a 19 year old
woman who smokes 25 cigarettes per day attending at 32 weeks gestation. You have
palpated her abdomen and the symphysiofundal height measured 26cm. Her blood
pressure is 120/70mmHg and there is no protein in her urine.
She reports that foetal movements are good and you find the foetal heart rate is
normal on auscultation. You diagnose intra-uterine growth restriction (IUGR) and refer
her for hospital consultation.

The patient attends and her consultant decides to regard this as a high-risk pregnancy
and to monitor the foetus closely. Apart from foetal heart monitoring, give 2 methods
by which foetal wellbeing might be assessed.
USS
Measure abdo circumference and BPD to confirm IUGR
Assess liquor volume
Doppler umbilical arterial MCA
Ductus venous Doppler
NOT Cordocentesis (used if a congenital infection is suspected) as its too invasive

On review of the hx, what advice would you give this patient to reduce the impact of
IUGR?
Smoking Cessation
The patient continues with monitoring to 37 weeks when she goes into spontaneous
labour.

Because the pregnancy is regarded as high-risk, the condition of the foetus during
labour is monitored with a foetal heart monitor (cardiotocograph/CTG). Give 2
examples of abnormalities you would look for on the foetal heart monitor trace which
would indicate a foetal problem.
Reduced Variability (<5bpm changes for >40mins)
Baseline Tachycardia
Late Decelerations
NOT Loss of Accelerations as can occur in normal pregnancy

What short-term (i.e. pregnancy and peri-natal period) consequences are there of
IUGR?
Respiratory Distress Syndrome
Low Birth Weight risk of perinatal mortality, still birth and Necrotising
Enterocolitis
Hypoglycaemia
Hypothermia

153

What 2 potential long-term medical complications may this baby be exposed to when
it becomes an adult?
Type II DM
Coronary Artery Disease
Cerebral Palsy
Mental Retardation

154

Question 127: Obstetrics/ Gynecology


36 year old woman presents to her GP as she is worried about her failure to become
pregnant. She and her husband have been trying for a baby for approximately 18
months with no success. Your initial history, examination and investigations lead you
to suspect endometriosis in the woman as a cause of the infertility.

Suggest 2 other symptoms that might be suggestive of endometriosis


Dysmenorrhoea (Secondary)
Dyspareunia
Dyschezia (difficulty defecating)
Chronic Pelvic Pain

Name 1 treatment option available for mild/moderate endometriosis, which will


increase increase the chances of conception.
Controlled Ovarian Hyper-stimulation e.g. Clomiphene Therapy
Surgical treatment of endometrial lesions (coagulation, diathermy, laser ablation) and
removal of endometriomas
This is most likely to the be the second point, but both are likely to get the mark apparently.

List 2 features you would find on biopsy of endometriosis.


Endometrial Glands
Endometrial Stroma
Evidence of Haemorrhage
Scarring or Fibrosis
Haemosiderin-Laden Macrophages

List 2 other causes of blockages of the Fallopian tubes apart from Endometriosis.
Hydrosalpinx (fluid blockage secondary to chlamydia)
PID or Previous Ectopic causing adhesion

In vitro fertilisation is proposed for the patient. List 3 issues that should be discussed
with the couple before a decision is made.
Success Rate
Alternatives e.g. Adoption
Consent to use fertilised ovum if relationship breaks up
Decision subject to psychological assessement and age
Limited attempts on NHS (2005 NICE guidelines state up to 3 cycles if couple is deemed
suitable)

155

Question 128: Obstetrics/ Gynecology


A 30 year old woman is sent by her GP to gynaecological outpatients complaining of
postcoital bleeding. She has never had a cervical smear. She is unmarried and has
had several different sexual partners.

You suspect cervical carcinoma. List 3 clinical signs you would look for on your initial
clinical examination.
Friable, red, raised area of cervix
Blood stained vaginal discharge
Pelvic mass
Lymphadenopathy

Colposcopic examination and biopsy are arranged and the histopathology confirms a
malignant tumour. List the 2 most common types of malignant tumour of the cervix.
SCC
Adenocarcinoma
The most common type of malignant tumour is often preceded by a premalignant
condition. Name the 3 stages of this premalignant condition in the order in which they
occur.
CIN I, II, III - ridiculous; we know.

What agent is important in causing the above premalignant condition?


HPV (16 and 18 = higher risk; 6 and 11 = lower risk)

Name 2 ways in which this agent can cause abnormal cell proliferation.
Integration of viral DNA into epithelial cell DNA encouraging abnormal proliferation
Viral antigens/proteins bind to host cell tumour suppressor genes inactivating them

156

Question 129: Obstetrics/ Gynecology


A 25-year old is seen in the A&E department with a history of right iliac fossa pain and
slight vaginal bleeding after 7 weeks of amenorrhoea.

What is the differential diagnosis? Give 2 conditions.


Ectopic Pregnancy
Miscarriage

Give 2 important points in the history which you would wish to elicit.
Onset and duration of pain i.e. acute or insidious
Has she had a positive pregnancy test?
Dyspareunia over previous few days (common in ectopic)
Any change in bowel habit? (diarrhoea can be present in ectopic or acut appendicitis)
Does she know if she is pregnant or not? (i.e. has she had a urine or blood CG test?)
Previous pregnancies any history of ectopic or recurrent miscarriages?

Give 2 investigations you would carry out.


Urinary pregnancy test
Bloods FBC, Clotting, hCG, Group and Save
Imaging Transvaginal USS

Name 1 treatment option for each of your possible diagnoses.


Ectopic Surgical e.g. salpingectomy/salpingotomy or Medical e.g. Methotrexate
Miscarriage surgical management of miscarriage (SMM new name for ERPC)

What injection must be given to a Rh -ve pregnant woman who bleeds during
pregnancy?
Anti-D immunoglobulin (normally given at 28-32wks once or twice and 72hrs after birth)

157

Question 130: Paediatrics


You are a paediatric SHO who is doing baby checks prior to hospital discharge of
newborn babies on day 2 of life.

In generic terms list two purposes of the new-born examination prior to discharge. Do
not list specific clinical problems.
To confirm normality so as to reassure the patients
To identify and act upon any abnormalities

Give 2 examples of anatomical conditions specific to the new-born that may be elicited
on the clinical examination check.
Developmental Hip Dysplasia
Scoliosis
Talipes
Craniosynostosis

You have to perform a Guthrie test. Briefly (less than 50 words) describe how you
would explain this to the parents.
It involves cleaning the heel then pricking the foot with a small needle to obtain a small
quantity of blood, which is collected on filter paper before being sent to the lab for analysis

List 2 tests that can be performed by the Guthrie test.


Guthrie Assay (identifies Phenylketonuria)
Blood Dihydropteridine Reductase Assay (identifies Phenylketonuria due to defective BH4
synthesis)
Immune Reactive Trypsin (Cystic Fibrosis)
Thyroid Function Tests (Hypothyroidism)
MCAD Deficiency (enzyme deficiency resulting in inability to perform gluconeogenesis from
fat making Hypoglycaemia very likely with any form of increased metabolic demand)
Sickle Cell Disease

Your consultant is undertaking a research project looking at vitamin levels using a


sample from the Guthrie test. He asks you to ask the Mother whether this can be done.
List 4 principles that you would discuss with the mother while gaining her approval to
undertake the project.
Explain the project in full including proposed benefit and any risks associated with additional
testing
Explain that it is entirely her decision and that she should not feel pressured in any way
Give her adequate time to consider
Allow her time to ask any questions she might have

158

Question 131: Obstetrics/ Gynecology & Paediatrics


You are dealing with a mother who is very anxious following a term birth. You are
undertaking a clinical examination at day 2 post-partum.

At the end of your clinical examination you find no abnormal findings apart from
jaundice. List 2 clinical reasons why jaundice is important in the newborn.
Risk of Acute and/or Chronic Bilirubin Encephalopathy (aka Kernicterus)
May be first indication of potentially treatable condition e.g. Hypothyroidism
May indicate serious underlying pathology e.g. Biliary Atresia, Zellwegers Syndrome

Which 2 particular patterns of jaundice are most clinically concerning and why?
Jaundice that appears within 24hrs
Jaundice lasting >2wks
Both would increase the likelihood of an underlying pathological cause

What treatment is used for jaundice in premature babies and how does this work?
Phototherapy
Causes photochemical reactions that transform bilirubin into isomers that:
Are less lipophilic and hence more easily excretable
Produce breakdown products that do not require conjugation in the liver

The mother has no nearby family support and is asking about help in the home after
discharge. Explain what community support is available immediately after discharge
and in the weeks that follow. List 4 mechanisms by which that support can be
delivered that does not include family or friends.
Community Midwife
Health Visitor
GP
Social Work

159

Question 132: Paediatrics


You are sitting in on a clinic during your paediatric attachment. A baby (Stewart) who
is now 6 months old is being reviewed as he was born prematurely at 29 weeks. His
mother is very anxious, as he is not reaching the same milestones as her best friends
baby who was born the same month at term.

Name 4 different milestones you would expect a 6 month old to exhibit.


Gross Motor Rolls from front to back
Fine Motor/Vision Palmar Grasp
Hearing/Speech Babbling/turning towards sounds
Social Enjoys bath and attempts to bring objects to mouth
Note: the Paediatrics Revision Guide given to us by the medical school states that by 6mo
children should be saying Ma/Da. This is incorrect.

What is the most likely reasons that Stewarts development is delayed?


Pre-term Birth

How would you explain this to his anxious mother and what is the prognosis?
Explain it clearly, concisely and at a level she can understand with multiple
opportunities for her to ask questions in a calm and quiet environment.
Prognosis Babies born earlier are at a higher risk of developmental delay, but
she is bringing Stewart to clinic for regular review so that we can detect this
earlier and hopefully minimise any difficulties faced.

On the way out the door she has one final question: what will Stewarts next
immunization be and when will he receive it?
Hib/Men C booster + MMR + Pneumococcal (3rd dose) at 12-13 months.

160

Question 133: Obstetrics/ Gynecology & Pediatrics


You are the JHO on call when a 25-year old mother 31 weeks into her second
pregnancy arrives in the emergency department after her waters broke. You suspect
she will be going into premature labour.

You are asked to speak with the mother regarding delivery. List 3 complications of
prematurity that would be relevant to a child of this age that you would discuss with
her.
Respiratory distress may need support with CPAP or oxygen therapy
Sepsis premature babies are more prone to infection
GI baby will not be able to suck and will require nasogastric tube feeds
More likely to get jaundiced and need phototherapy
Bleeding increased risk of intraventricular haemorrhage
PDA increased risk

At four hours of age, this baby continues to have signs of respiratory distress. List 4
signs that you would look for in a child with respiratory distress.
RR
Nasal Flaring
Accessory Muscle Use
Intercostal Indrawing
Grunting
Tracheal Tug
Sunken Fontanelle
Head Bobbing
Cyanosis
Stridor
Stertor
Inability to lie down or cry
Apnea
Agitation

List 4 investigations you would consider relevant to your management of this child.
Pulse Oximetry
FBC
CXR
Blood Cultures
Blood Gases

List 4 treatments that you think would be worth considering in this case.
Oxygen Therapy
CPAP
Intubation
Antibiotics to cover for sepsis
In real life youd consider these for about 2 seconds before remembering youre an FY1 and
so, should CALL A SENIOR

161

Question 134: Psychiatry


A 55 year old woman presents to you in the Psychiatry Outpatients with symptoms of
low energy and depression. Her GP explains in the letter that that she is feeling
hopeless and has considered killing herself.

Hopelessness is the biggest predictor of a single behaviour in depression. What is


that behaviour?
Attempted Suicide

On examination her pulse is slowed and she has possible evidence of a goitre. What
single blood test would you undertake and why?
TFTs exclude or confirm hypothyroidism as the cause of her symptoms
The blood test comes back within the normal range. Other blood tests and physical
examination are all normal. After consultation with your senior colleagues, you decide
to prescribe Fluoxetine, an SSRI medication.

What advice would you give her about the time it will take for the Fluoxetine to begin
to work?
SSRIs typically take between 2-6wks before their effect is felt
3-4wks is sometimes the clinical figure quoted and although this wouldnt be marked wrong
2-6wks is better.

She responds to this medication. How long should she continue to take it once she
feels better?
At 6-9 months (from Maudsley guidelines)

After three months on the medication, her depression worsens. She wishes to
continue the Fluoxetine. Suggest two medication strategies that can be offered to
boost the effectiveness of her tablet.
Increase the dosage and check compliance
Add a mood stabiliser e.g. Lithium
Add a dopamine re-uptake inhibitor e.g. Bupropion (rarely done in clinical practice)
Add a 5-HT2 receptor antagonist e.g. Mirtazapine (beware serotonin syndrome)

162

In spite of these changes, her depression continues to worsen. You decide to change
her medication to the SNRI, Venlafaxine. How long a washout is recommended in BNF
before this can be done?
Current Maudsley guidelines state that there is no need for a withdrawal period and that
Venlafaxine can be started immediately after cessation of Fluoxetine.
Fluoxetine/MAOIs Venlafaxine = 2 weeks Paroxetine/Citalopram Venlafaxine =
3 days
TCAs Venlafaxine = 4 days Moclobemide Venlafaxine = 1 day
This question is absolute guff. As anyone who has actually looked in the BNF will tell you,
there is no specific figure given for the washout period; all the BNF has to say is that there
should be a washout period. Depending on the resource used, the washout varies from 02wks. The figures i are from a set of Bristol guidelines recommend to us by a psychiatric
trainee and may be considered correct. However, the consultant has recommended we
answer using the text in red as you couldnt be marked wrong for using guidelines,
Unfortunately the change is not effective. She becomes acutely suicidal and says that
she is going to kill herself.

Is it necessary to complete a Section of the Mental Health Act to prevent her leaving
and could you legally detain her in the surgery if this was not completed in time?
Yes: under common law nurses in NHS properties may detain someone for 2 hours without
completing a form with a further 1 hour given for a doctor to examine the patient. If
necessary, an Emergency Detention Certificate (72hrs detention) form can then be
completed.
In practice, you would let them go, call the police to find them then complete an EDC while
theyre brought in.

163

Question 135: Psychiatry


A 24 year old woman presents to casualty feeling lightheaded, short of breath and with
tingling in her fingers. Physical assessment (including ECG) is unremarkable apart
from moderate tachycardia, hyperventilation and tremor. She gave a history of several
similar episodes occurring when in supermarkets or on buses. You believe she may
be having a panic attack.

List 4 additional symptoms that would support a psychological cause for her problem.
Globus Pharyngis
Fear of Dying
Collapse
Blackout
Chest Tightness despite no identifiable cardiac abnormality
NOT Clear Trigger as this is not the case with panic attacks

What is the most likely underlying diagnosis?


Panic Disorder with Agoraphobia
NOT phobic anxiety disorder.
Panic Attack might be correct but given that its mentioned above/ the question is asking for
the most likely underlying diagnosis, the answer they probably want is Panic Disorder with
Agoraphobia

What is the mechanism for the tingling in her fingers?


Hyperventilation respiratory alkalosis paraesthesia

What simple non-pharmacological measures might you take immediately in casualty


to treat the acute problem?
Encourage patient to breathe deeply e.g. 7 seconds in and 11 out
Challenge their negative thoughts e.g. Youre not going to die.
DO NOT immediately take patient into a quiet room and remain very calm around them as
this simply continues the negative cycle and reinforces their fear
NOT encourage patient to breathe through a paper bag as this is no longer recommended.
Instead, you encourage them to breathe as described above.

Give 2 drug treatments which may be appropriate for short-term management.


-Blockers e.g. Propranolol
SSRIs e.g. Escitalopram
NOT Bupropion
NOT Benzodiazepines - this is considered exceptionally bad practice in primary care.

What psychological treatment is appropriate for longer-term management?


Anxiety Management
164

Though other therapies (e.g. Cognitive Behaviour Therapy) are relevant they are not the best

165

Question 136: Psychiatry & Neurology


Mr B is a 54 year old surgical in-patient who had an emergency operation 2 days ago
following perforation of a duodenal ulcer. At 2.00am you are called by the night staff
who report that he is v agitated, confused and trying to leave the ward. They also tell
you that he lives alone and has no close relatives. You think that delirium (acute
confusional state) is an important diagnosis to consider.

Suggest 3 psychiatric symptoms or signs you would look for, either from the above
presentation or from further assessment, to confirm your diagnosis.
Reversal of Sleep/Wake Pattern
Disturbance of Cognition
Emotional Changes
Visual/Auditory Hallucinations
Delusions
Loss of Insight (disorientation)

List 4 possible predisposing causes which could have precipitated this attack of
delirium.
Recent Surgery risk of infection
Recent Surgery risk of vascular event
Recent Surgery risk of metabolic imbalance
Recent Surgery post-op opioids

List 3 cognitive functions you would like to test and give an example of one question
for each function.
Orientation to Time, Place and Person
Recall i.e. list 3 unconnected objects and ask to repeat back immediately then later (delayed
recall)
Language i.e. name an object e.g. pen

166

Question 137: Psychiatry & Neurology


A 74 year old woman is admitted to hospital having collapsed at home. Her daughter
tells you that she has been drowsy for the past couple of days and has had a cough
productive of green sputum. She smokes 20 cigarettes per day. On examination she is
centrally cyanosed and you find her temperature to be 39.5. Her pulse rate is 120
beats/minute and the blood pressure is 130/80mmHg. Shes noticed to be choking on
food. She has no ankle oedema. She is restless and vague about recent events and
has abbreviated mental test score of 6/10.

What is the diagnosis of this ladys acute deterioration in mental dysfunction?


Delirium

Give 2 possible causes of this ladys deteriorating mental function.


Infection (e.g. Pneumonia)
Hypoxia
Cerebrovascular Accident

You undertake an arterial blood gas estimation. What 2 gas abnormalities would you
predict that would relate to the above condition?
Hypoxia
Acidosis

How would you try to correct these gas abnormalities and what precautions should
you take?
Oxygen therapy
Use low flow i.e. 24% Venturi Mask due to risk of COPD in this patient

List two antibiotics that would be most appropriate for treatment of the underlying
condition
Amoxicillin
Clarithromycin

How would these best be administered given the present situation?


Intravenous
Twelve hours following admission, the staff nurse tells you that Mrs Smith has not
passed urine.

What would be the most appropriate investigation and how would you correct this
abnormality?
Investigation U+E or USS (Pelvic) Weve been told this could be either
Urinary Catheter

167

Question 138: Psychiatry


A 25 year old man presents at your surgery complaining of feeling stressed at work,
he describes feeling useless and incapable and has no interest in his usual social
activities. He had recently moved out of home to live with his girlfriend but this
relationship has just ended.

List 4 potential diagnosis?


Adjustment Disorder
Schizophrenia (-ve symptoms)
Organic Disorder e.g. Hypothyroidism
Substance Abuse
Dysthymia
Depressive Illness (despite the fact that this is mentioned later, weve been told its still valid)

List 2 biological (i.e. somatic) symptoms of depression.


CALS:
Concentration
Appetite
Libido
Sleep
You think Mr J is suffering from depression and wish to commence him on
antidepressants.

What two possible groups of antidepressants could be used and their mechanisms of
action
SSRIs inhibit 5-HT re-uptake 5-HT in synaptic cleft receptor binding
mood
SNRIs similar to the above mechanism but also inhibits Noradrenaline reuptake mood
TCAs essentially act like SNRIs but also block Ca/Na channels toxicity in
Overdose
A review appointment is made for 1/52 but unfortunately Mr J is admitted to hospital
with an attempted suicide. His mother finds him unconscious at home. It appears he
has taken an overdose of coproxamol and has left a suicide note. If his mother had not
called by to check on him he may not have been found until the following day. On
examination he has pinpoint pupils.

What specific antidote should be given?


N-Acetyl Cysteine

What other complication may develop later and should be anticipated?


Liver Failure

Give two features that are in favour of this being a serious attempt?
Left a suicide note
Attempt made when likely not to be interrupted

168

169

Question 139: Psychiatry


A 54-year old lady who lives alone presents with overactivity, pressure of speech and
flight of ideas. She has neglected her hygiene and house. The Environmental Health
Officer has been called in to fumigate the house, which is covered in dog excrement
and rat infested. You suspect that she has hypomania and decide to treat her acutely
with haloperidol, and in the longer term with lithium carbonate.

You discover her husband had died 18 months earlier after a lingering cancer. How
may this be relevant to the current picture? List 3 points?
She may be undergoing a grief reaction
She may be undergoing an adjustment reaction
He may have been her primary carer and hence, may have prevented her from deteriorating
previously by helping her/encouraging her to seek help

List 3 non-drug factors that need to be put in place by the Care Programme
Approach once treatment has been stabilised.
Community Psychiatric Nurse Review
OP Review
Home Care Assessment
Social Care

Six months after discharge, she suffers an accidental overdose of the lithium she has
been prescribed to treat her maintenance therapy. What 2 investigations would you
institute to monitor the situation?
Urea and Electrolytes (renal failure)
Lithium Levels

What is the main principle of therapy for the acute management of this overdose?
Supportive Therapy e.g. IV Fluids, Cardiac Monitoring etc.

170

Potrebbero piacerti anche